Comprehensive Review in Clinical Neurology: A Multiple Choice Question Book for the Wards and Boards

Chapter 5. Adult and Pediatric Epilepsy and Sleep

Question

1. If an AED is required in pregnancy, the safest choice of the following would be:

a. Phenytoin

b. Primidone

c. Lamotrigine

d. Valproic acid

e. Phenobarbital

2. Which of the following AEDs is associated with weight loss?

a. Pregabalin

b. Gabapentin

c. Topiramate

d. Carbamazepine

e. Valproic acid

3. A 36-year-old male with a prior history of HSV infection presents with new-onset epilepsy that you suspect is due to his prior encephalitis affecting the temporal lobes. The clinical features of seizures in mesial temporal lobe epilepsy may have all of the following characteristic symptoms, except:

a. Olfactory hallucinations

b. Altered consciousness

c. Complex partial seizures

d. Automatisms

e. Tonic posturing of one limb (fencer’s posture)

4. Which of the following AEDs would have the least drug–drug interactions?

a. Gabapentin

b. Carbamazepine

c. Valproic acid

d. Lamotrigine

e. Phenytoin

5. In a young child with generalized epilepsy refractory to multiple antiepileptic medications, what would be the next best choice of treatment, if tolerated?

a. Corpus callosotomy

b. Ketogenic diet

c. Vagus nerve stimulation

d. Phenytoin

e. Carbamazepine

6. Which of the following is the best treatment option for simple febrile seizures?

a. IV lorazepam

b. Rectal diazepam

c. Supportive management

d. Phenobarbital

e. Intranasal midazolam

7. Which of the following mutations has not been associated with generalized epilepsy with febrile seizures plus (GEFS+)?

a. SCN1D

b. SCN1A

c. SCN1B

d. SCN2A

e. GABRD

8. A 13-year-old girl is being evaluated for epilepsia partialis continua. Rasmussen’s syndrome is suspected. What would be the most common finding on brain MRI in Rasmussen’s syndrome?

a. Lissencephaly

b. Schizencephaly

c. Cortical atrophy

d. Pachygyria

e. Porencephaly

9. A 12-year-old boy presents to your office with a history of progressively worsening frequency and severity of daily myoclonic seizures. His mitochondrial testing has so far been negative, although you suspect a progressive myoclonic epilepsy (PME) of some type. What would be the best antiepileptic medication to try first, given there are no contraindications?

a. Carbamazepine

b. Phenytoin

c. Oxcarbazepine

d. Valproic acid

e. Gabapentin

10. The benefits of fosphenytoin over phenytoin include all of the following, except:

a. Can be given intramuscularly

b. Less cardiovascular side effects

c. Achieves therapeutic plasma concentrations faster

d. Faster rate of IV administration possible

e. Less infiltration reactions (purple glove syndrome)

11. Which of carbamazepine the following has not been associated with worsening of myoclonic seizures?

a. Topiramate

b. Carbamazepine

c. Lamotrigine

d. Pregabalin

e. Vigabatrin

12. Which of the following antiepileptic medications is a hepatic enzyme inhibitor?

a. Phenytoin

b. Carbamazepine

c. Valproic acid

d. Phenobarbital

e. Primidone

13. Which of the following EEG findings would be associated with the highest incidence of seizures?

a. Small sharp spikes

b. 6-Hz spike and wave

c. Wicket spikes

d. 3-Hz spike and wave

e. 14 and 6 positive spikes

14. At what age do human beings attain the predominant α-frequency (posterior background) that is seen in adults?

a. 6 to 8 years

b. 8 to 10 years

c. 10 to 12 years

d. 12 to 14 years

e. 14 to 16 years

15. A 47-year-old woman presents with confusion, fever, and seizures. CSF studies are positive for HSV infection. What would be the most likely finding on an EEG in this patient?

a. Triphasic waves

b. Wicket spikes

c. Periodic lateralized epileptiform discharges (PLEDs)

d. Polyspikes

e. Fast spike–wave complexes

16. In a patient with absence epilepsy, absence status epilepticus can be precipitated by all of the following antiepileptic medications, except:

a. Phenytoin

b. Topiramate

c. Carbamazepine

d. Lamotrigine

e. Gabapentin

17. Which of the following antiepileptic medications is the most likely to have an effect on steroid hormone concentration in the blood in patients taking oral contraceptive pills and, therefore, lead to contraceptive failure?

a. Levetiracetam

b. Gabapentin

c. Topiramate (dose <200 mg/day)

d. Oxcarbazepine

e. Zonisamide

18. An 8-year-old boy is brought to your office by his parents. They tell you that his teacher suggested the boy should see a neurologist for inattention. His EEG is shown in Figure 5.1 and is characteristic of what type of epilepsy?

FIGURE 5.1 EEG (Courtesy of Dr. Andreas Alexopoulos)

f. Juvenile myoclonic epilepsy (JME)

g. Absence epilepsy

h. Myoclonic epilepsy

i. Benign childhood epilepsy with centrotemporal spikes (benign rolandic epilepsy of childhood)

j. Lennox–Gastaut epilepsy

Questions 19–20

19. A 12-year-old boy with complaints of early morning falls, clumsiness, and dropping objects presents with a generalized tonic–clonic (GTC) seizure. His EEG shown in Figure 5.2 suggests which of the following?

FIGURE 5.2 EEG (Courtesy of Dr. Andreas Alexopoulos)

a. Juvenile myoclonic epilepsy (JME)

b. Absence epilepsy

c. Myoclonic epilepsy

d. Benign childhood epilepsy with centrotemporal spikes (benign rolandic epilepsy of childhood)

e. Lennox–Gastaut epilepsy

20. Which of the following antiepileptic drugs would typically be the first-line treatment for this patient?

a. Carbamazepine

b. Phenytoin

c. Phenobarbital

d. Valproic acid

e. Topamax

Questions 21–22

21. A 7-year-old boy has recurrent nocturnal events, which awaken his parents, because they hear “his bed shaking.” They have witnessed him “shaking uncontrollably” on entering his bedroom on occasion. They have also seen only his face and arm twitching before or after the convulsions. His EEG is shown in Figure 5.3 and suggests which of the following?

FIGURE 5.3 Interictal EEG (Courtesy of Dr. Andreas Alexopoulos)

a. Juvenile myoclonic epilepsy (JME)

b. Metabolic encephalopathy

c. Myoclonic epilepsy

d. Benign childhood epilepsy with centrotemporal spikes (benign rolandic epilepsy of childhood)

e. Periodic lateralized epileptiform discharges (PLEDs)

22. When necessary, what is the antiepileptic of choice in this condition, given no contraindications?

a. Phenytoin

b. Valproic acid

c. Topiramate

d. Carbamazepine

e. Ethosuximide

23. A 56-year-old man presents with confusion and seizures. His EEG is shown in Figure 5.4 and is characteristic of which of the following?

FIGURE 5.4 EEG (Courtesy of Dr. Andreas Alexopoulos)

a. Juvenile myoclonic epilepsy (JME)

b. Metabolic encephalopathy

c. Myoclonic epilepsy

d. Benign childhood epilepsy with centrotemporal spikes (benign rolandic epilepsy of childhood)

e. Periodic lateralized epileptiform discharges (PLEDs)

Questions 24–25

24. A 6-month-old baby boy is brought to your office by his parents. They describe sudden tonic flexion of limbs and body occurring in clusters after waking. His EEG is shown in Figure 5.5. What is your diagnosis?

FIGURE 5.5 EEG (Courtesy of Dr. Andreas Alexopoulos)

a. Juvenile myoclonic epilepsy (JME)

b. Hypsarrhythmia of infantile spasms

c. Myoclonic epilepsy

d. Benign childhood epilepsy with centrotemporal spikes (benign rolandic epilepsy of childhood)

e. Absence epilepsy

25. Which of the following is the most accepted treatment for this disorder?

a. Lamotrigine

b. Phenytoin

c. Phenobarbital

d. ACTH

e. Carbamazepine

Questions 26–28

26. Regarding the pharmacokinetics of phenytoin, which of the following is correct?

a. It is predominantly renally metabolized

b. It is exclusively metabolized by the liver

c. It has zero-order (nonlinear) kinetics

d. It is a hepatic enzyme inhibitor

e. It has first-order (linear) kinetics

27. Which of the following is not a common long-term side effect of chronic phenytoin use?

a. Hirsutism

b. Osteoporosis

c. Gingival hyperplasia

d. Cortical atrophy

e. Coarse facial features

28. A patient with known epilepsy on phenytoin presented to the emergency department with a breakthrough seizure. His total phenytoin level is 10 μg/mL, but when you look back at his prior levels, it is usually around 15 μg/mL. Assuming a volume of distribution of 0.8 L/kg, which of the following would be the best IV dose to boost him to his baseline level if he weighs 75 kg (assuming that the patient does not have any comorbidities that are known to modify phenytoin kinetics, such as uremia or hypoalbuminemia)?

a. 1000 mg

b. 600 mg

c. 300 mg

d. 150 mg

e. 50 mg

Questions 29–31

29. A 43-year-old male on valproic acid for long-standing epilepsy presents to the emergency department with a breakthrough seizure. He is 70 kg and his total valproic acid level is 70 μg/mL, although he normally has levels around 100 μg/mL. Assuming a volume of distribution of 0.2 L/kg, what will be your correcting IV bolus?

a. 120 mg

b. 220 mg

c. 320 mg

d. 420 mg

e. 520 mg

30. Which of the following is not a possible long-term side effect of valproic acid ?

a. Hair loss

b. Cerebellar atrophy

c. Polycystic ovarian syndrome

d. Tremor

e. Weight gain

31. The neurologist is called to see this patient as a consult in the emergency department. The emergency department physician asks if lamotrigine could be added as an additional antiepileptic agent to his valproic acid. Which of the following best describes the interaction between these two medications?

a. Lamotrigine significantly increases the half-life of valproic acid

b. Valproic acid significantly decreases the half-life of lamotrigine

c. Valproic acid significantly increases the half-life of lamotrigine

d. Lamotrigine significantly decreases the half-life of valproic acid

e. Lamotrigine and valproic acid do not have any significant interaction

Questions 32–35

32. Which of the following is not a known side effect of carbamazepine?

a. Dizziness

b. Nystagmus

c. Drowsiness

d. Hypernatremia

e. Nausea

33. Which of the following is true regarding carbamazepine?

a. It inhibits its own metabolism

b. For new-onset, frequent seizures, carbamazepine is a good option as initial antiepileptic therapy

c. It has no effect on its own metabolism

d. It induces its own metabolism

e. It has no hepatic metabolism

34. A 34-year-old man being treated with carbamazepine presents with dizziness and nystagmus after valproic acid was recently added for his epilepsy. These symptoms are likely related to:

a. Elevated levels of carbamazepine

b. Elevated levels of valproic acid

c. Elevated levels of 10,11-carbamazepine epoxide

d. Withdrawal symptoms from increased carbamazepine metabolism

e. Valproic acid itself, as his body adjusts to a second antiepileptic

35. Oxcarbazepine is a structural derivative of carbamazepine. Which of the following is true about oxcarbazepine in comparison to carbamazepine?

a. Is not a hepatic enzyme inducer

b. No risk of hyponatremia

c. Is not metabolized to an epoxide

d. Indicated in both partial and generalized epilepsy

e. No risk of rash as seen with carbamazepine

36. Which of the following is a mechanism of action of benzodiazepines?

a. Chloride channel antagonism

b. Chloride channel agonism

c. GABAA antagonism

d. GABAA agonism

e. GABAB agonism

Questions 37–38

37. Which of the following is the least likely side effect of lamotrigine?

a. Stevens–Johnson syndrome

b. Blurred vision

c. Ataxia

d. Dizziness

e. Cognitive complaints

38. Which of the following is incorrect regarding lamotrigine and hormonal interactions?

a. Oral contraceptives containing only ethinylestradiol increase lamotrigine clearance

b. Oral contraceptives containing both ethinylestradiol and progesterone increase lamotrigine clearance

c. Oral contraceptives containing only progesterone increase lamotrigine clearance

d. Lamotrigine clearance is increased significantly during pregnancy

e. Hormone replacement therapy increases lamotrigine clearance

Questions 39–40

39. Which of the following is false regarding topiramate?

a. It is metabolized predominantly by the liver

b. It is a sodium channel antagonist

c. It is an NMDA-glutamate antagonist

d. It is a GABAA agonist

e. It is a carbonic anhydrase inhibitor

40. Which of the following is not a side effect of topiramate?

a. Word-finding difficulty

b. Increased appetite

c. Paresthesias

d. Kidney stones

e. Acute angle-closure glaucoma

41. Which of the following is incorrect regarding the mechanism of action of lacosamide?

a. Selectively enhances fast inactivation of voltage-dependent sodium channels

b. Stabilizes hyperexcitable neuronal membranes

c. Inhibits repetitive neuronal firing

d. Enhances slow inactivation of voltage-dependent sodium channels

e. Binds to the collapsin response mediator protein 2 (CRMP2)

42. Which of the following is incorrect regarding rufinamide?

a. It modulates activity at neuronal sodium channels

b. It is FDA-approved as an adjunct for Lennox–Gastaut syndrome

c. It prolongs the inactive state of neuronal sodium channels

d. Elimination of rufinamide occurs primarily through renal excretion

e. It is metabolized by the cytochrome P450 system

43. A 21-year-old man comes to the clinic because he has been having spells in which he suddenly stops what he was previously doing and stares for about a minute, sometimes picking at his nose and his shirt. He cannot recall what happens during the spell itself. He says, however, that he knows when a spell is going to happen because he experiences a warm sensation in his epigastric region, followed by a sensation of fear and a rapid recollection of episodes of past life experiences along with palpitations. He may have postictal confusion. His EEG shows focal spikes. Which of the following best describes the type of seizure this patient has?

a. Temporal lobe seizures

b. Frontal lobe seizures

c. Absence seizures

d. Occipital lobe seizures

e. Parietal lobe seizures

44. An 18-year-old man presents for a second opinion regarding seizures, which occur multiple times per day, starting with eye deviation to the left. He then emits a loud “cry-like” sound, his head turns left, and his left arm adopts a tonic posture with shoulder external rotation and abduction with arm extension, while his right arm is flexed. These seizures last for about 30 seconds, and sometimes generalize. Which of the following is the most likely origin of his seizures?

a. Right temporal lobe

b. Right supplementary motor area

c. Left temporal lobe

d. Left supplementary motor area

e. Right parietal lobe

45. A 5-month-old boy with developmental delay is brought for evaluation of repeated clusters of spasms characterized by bilateral arm flexion along with neck, trunk, and leg flexion. His EEG is shown in Figure 5.5. Which of the following is incorrect regarding this condition?

a. MRI should be done in every patient with this condition

b. Diagnosis of tuberous sclerosis should be considered and ruled out

c. ACTH is used for treatment

d. These types of seizures do not need treatment and the prognosis is good

e. Vigabatrin may be effective to treat this condition

46. A 2-month-old girl has blindness, infantile spasms, and an abnormal retinal examination. Her brain MRI shows agenesis of the corpus callosum. Which of the following is the most likely cause?

a. West syndrome

b. Ohtahara syndrome

c. Severe myoclonic epilepsy of infancy (Dravet syndrome)

d. Aicardi syndrome

e. Myoclonic–astatic epilepsy (Doose syndrome)

47. A 4-year-old boy with normal cognitive development is brought in after a generalized tonic-clonic (GTC) seizure. Over the past couple of months he has been having episodes of falls, suffering multiple injuries. His mother describes episodes in which he loses tone, causing him to fall, and episodes of rapid “jerk-like” symmetric movements of the upper extremities that may also lead to falls. There is no clear loss of consciousness with these brief events. His interictal EEG shows parietal rhythmic θ and bilateral synchronous irregular 2- to 3-Hz spike and wave complexes. Which of the following is the most likely diagnosis?

a. Severe myoclonic epilepsy of infancy (Dravet syndrome)

b. Ohtahara syndrome

c. Benign myoclonic epilepsy of infancy

d. Generalized epilepsy with febrile seizures plus (GEFS+)

e. Myoclonic–astatic epilepsy (Doose syndrome)

48. A 2-year-old boy with a history of developmental delay and a prolonged febrile seizure at age 1 is admitted with frequent seizures of multiple types. He initially began having myoclonic seizures; however, he has now also developed absence seizures, as well as unilateral and generalized tonic-clonic (GTC) seizures. Which of the following is the most likely diagnosis?

a. Severe myoclonic epilepsy of infancy (Dravet syndrome)

b. Ohtahara syndrome

c. Benign myoclonic epilepsy of infancy

d. Generalized epilepsy with febrile seizures plus (GEFS+)

e. Myoclonic–astatic epilepsy (Doose syndrome)

49. You see a 15-day-old baby with severe hypotonia and frequent tonic spasms occurring in clusters of more than 100 times/day. The EEG shows burst suppression, present when the patient is either awake or asleep. Which is the most likely diagnosis?

a. Severe myoclonic epilepsy of infancy (Dravet syndrome)

b. Ohtahara syndrome

c. Benign myoclonic epilepsy of infancy

d. Generalized epilepsy with febrile seizures plus (GEFS+)

e. Myoclonic–astatic epilepsy (Doose syndrome)

50. A 2.5-year-old child comes for a follow-up appointment. At the age of 8 months he began having brief generalized myoclonic seizures, associated with fast (>3-Hz) spike–wave and polyspike and wave discharges on the EEG. Interictal EEG is normal. His seizures are well controlled on valproic acid and his development has been normal. Which of the following is the most likely diagnosis?

a. Severe myoclonic epilepsy of infancy (Dravet syndrome)

b. Ohtahara syndrome

c. Benign myoclonic epilepsy of infancy

d. Generalized epilepsy with febrile seizures plus (GEFS+)

e. Myoclonic–astatic epilepsy (Doose syndrome)

51. A newborn is being evaluated for seizures, which are characterized by apneic spells associated with unilateral or bilateral clonic movements. Starting on day 5 of life he has been having multiple spells per day. His neurologic examination is otherwise normal in between seizures. His interictal EEG is normal. Which of the following is the most likely diagnosis?

a. West syndrome

b. Benign neonatal seizures

c. Aicardi syndrome

d. Ohtahara syndrome

e. Benign myoclonic epilepsy of infancy

52. A 4-year-old boy presents for evaluation of spells. Apparently, the episodes begin with some visual phenomena, which he cannot describe, followed by eye deviation and vomiting. He has had a total of three of these spells. His EEG shows a normal background with high-voltage occipital spikes, which disappear with eye opening. Which of the following is the most likely diagnosis?

a. Ohtahara syndrome

b. Late-onset or Gastaut-type childhood occipital epilepsy

c. Early-onset or Panayiotopoulos-type childhood occipital epilepsy

d. Dravet syndrome

e. Doose syndrome

53. A 3-year-old boy with mental retardation is brought for evaluation of his seizures. He began having drop attacks about a year ago, but progressively has developed multiple seizure types, including absences, tonic seizures, and clonic seizures. Multiple antiepileptic agents have been tried with mild improvement, but he still has multiple seizures per day. His EEG shows 2-Hz spike–wave discharges. Which of the following is the most likely diagnosis?

a. Panayiotopoulos syndrome

b. West syndrome

c. Landau–Kleffner syndrome

d. Lennox–Gastaut syndrome

e. Seizures associated with mesial temporal lobe sclerosis

54. The parents of a 5-year-old boy report that he seems to be more withdrawn over the past several months. One year ago he began having seizures, initially myoclonic seizures and later generalized tonic-clonic (GTC) seizures. About 9 months ago he was noticed to have some problems understanding verbal communication, and he now appears to be aphasic. His EEG shows multifocal spikes. Which of the following is the most likely diagnosis?

a. Panayiotopoulos syndrome

b. West syndrome

c. Landau–Kleffner syndrome

d. Lennox–Gastaut syndrome

e. Seizures associated with mesial temporal lobe sclerosis

55. An 11-year-old girl is brought to the epilepsy monitoring unit for evaluation of possible “pseudoseizures.” Her spells occur only at night, and are described as large-amplitude and violent movements of all four limbs and her trunk. Given the hyperkinetic bizarre movements and a normal awake EEG, her seizures were thought to be nonepileptic by a local neurologist. In the epilepsy monitoring unit a seizure is captured during non-REM sleep. Which of the following is the most likely diagnosis?

a. Electrical status epilepticus during sleep (ESES)

b. Lennox–Gastaut syndrome

c. Landau–Kleffner syndrome

d. Autosomal dominant nocturnal frontal lobe epilepsy (ADNFLE)

e. Panayiotopoulos syndrome

Questions 56–59

56. A 15-year-old girl has seizures that begin with eye and head deviation toward the left, with subsequent generalization. Where is the most likely location of the seizure focus?

a. Hypothalamus

b. Right frontal lobe

c. Left frontal lobe

d. Right temporal lobe

e. Left temporal lobe

57. An 18-year-old man has seizures that begin with asymmetric tonic posturing in which his right arm is extended at the elbow with the fist clenched, while the left arm is flexed at the elbow. He subsequently has generalized tonic-clonic (GTC) seizures. Where is the most likely location of the seizure focus?

a. Right temporal lobe

b. Left temporal lobe

c. Right supplementary motor area

d. Left supplementary motor area

e. Right occipital lobe

58. A 32-year-old woman has seizures in which her left arm becomes dystonic, while she exhibits automatisms with the right arm. Where is the most likely location of the seizure focus?

a. Right temporal lobe

b. Left temporal lobe

c. Left frontal lobe

d. Hypothalamus

e. Left supplementary motor area

59. A 7-year-old boy has seizures that are characterized by uncontrollable episodes of laughter. Where is the most likely origin of his seizures?

a. Hypothalamus

b. Right frontal lobe

c. Left frontal lobe

d. Right temporal lobe

e. Left temporal lobe

60. A 56-year-old woman has a history of coronary disease, atrial fibrillation, and seizures secondary to a stroke she suffered last year, which are well controlled on phenytoin. Two weeks ago she was diagnosed with a urinary tract infection and was placed on trimethoprim–sulfamethoxazole and fluconazole. She is now admitted with unsteady gait and frequent falls. On examination she is lethargic and dysarthric, with nystagmus and ataxia. Which of the following tests you should order first?

a. MRI of the brain

b. EEG

c. Free and total phenytoin level

d. Urine culture

e. CT of the brain

Question 61–62

61. A 2-year-old girl without a significant past medical or family history has a generalized seizure lasting 5 minutes in the setting of a fever of 39°C. The patient recovered without any residual neurologic deficit. The mother would like to know the risk of recurrence. Which of the following is not a predictor of recurrence of febrile seizures (FS)?

a. Family history of FS

b. Age younger than 18 months at the time of FS

c. Shorter duration of fever prior to FS

d. Lower peak temperature at the time of FS

e. Complex FS

62. In a patient who suffered a febrile seizure (FS), which of the following is not a risk to develop subsequent epilepsy?

a. Family history of FS

b. Complex FS

c. Developmental delay

d. Family history of epilepsy

e. Neurologic abnormality

63. A 14-year-old boy is brought to the clinic for evaluation. He has had stimulus-sensitive myoclonus noticed about 4 years ago, which has become more frequent lately. Over the past 3 months he has been having generalized tonic-clonic (GTC) seizures, and he is clumsier and having frequent falls and problems with hand coordination. The neurologic examination demonstrates ataxia. His MRI is unremarkable and the EEG shows generalized spikes and waves. Genetic testing demonstrated an EPM1 gene mutation. Which of the following is the most likely diagnosis?

a. Lafora body disease

b. Unverricht Lundborg syndrome

c. Sialidosis

d. Juvenile myoclonic epilepsy

e. Neuronal ceroid lipofuscinosis

64. A 12-year-old boy with a history of migraines has short stature, ataxia, proximal weakness, mild cognitive impairment, deafness, and various types of seizures. He is admitted after a viral illness, becomes dehydrated, and is found to have lactic acidosis. He has been diagnosed with a progressive myoclonic epilepsy (PME); a muscle biopsy has been obtained and shown in Figure 5.6. Which of the following is a feature of this patient’s condition?

FIGURE 5.6 Muscle specimen (Courtesy of Dr. Richard A. Prayson). Shown also in color plates

a. Mutation affecting cystatin B

b. Cherry red spot on the fundoscopic examination

c. This is a mitochondrial disorder

d. EPM2 A mutation

e. Lafora bodies on skin biopsy

65. A 21-year-old man presents with progressive myoclonic jerks and generalized tonic-clonic (GTC) seizures. He has also had a mild and gradual onset of gait instability, ataxia, hyperreflexia, and decreased visual acuity, which is worse at night. His fundoscopic examination shows a cherry red spot. Which of the following is the most likely diagnosis?

a. Unverricht Lundborg syndrome

b. Lafora body disease

c. Sialidosis

d. Juvenile myoclonic epilepsy

e. Neuronal ceroid lipofuscinosis

66. A 14-year-old boy with progressive cognitive decline and ataxia has a history of myoclonic epilepsy and multiple seizure types. His EEG shows spikes and waves with predominance in the occipital region. A diagnosis was made on the basis of skin biopsy, which showed periodic-acid-Schiff (PAS)-positive intracellular inclusions. Which of the following is the most likely finding in this patient?

a. Mutation affecting cystatin B

b. Cherry red spot on fundoscopic examination

c. Ragged red fibers on muscle biopsy

d. EPM1 mutation

e. EPM2 A mutation

67. A 12-year-old boy with intractable epilepsy is being evaluated for surgical treatment of Rasmussen’s encephalitis. He has progressive cognitive decline, left hemiparesis, left visual field defect, and the EEG shows status epilepticus arising from the right hemisphere. Which of the following is incorrect regarding this condition?

a. There are autoantibodies against the glutamate receptor subunit GluR3

b. The seizures are typically well controlled with AED monotherapy

c. Histopathology shows perivascular cuffs of lymphocytes and monocytes, as well as glial nodules in the gray and white matter

d. There is spongy tissue degeneration in the long term

e. Hemispherectomy is a treatment option

68. Regarding the use of AEDs in the elderly, all the following statements are correct, except:

a. The distribution of hydrophilic drugs decreases

b. The distribution of lipophilic drugs decreases

c. Hepatic blood flow, bile flow, and protein synthesis decrease along with hepatic metabolism

d. Renal blood flow and glomerular filtration rate decrease

e. Gastric acidity may increase, making weakly basic drugs easily absorbed, and weakly acidic drugs less easily absorbed

69. An 18-year-old woman is diagnosed with juvenile myoclonic epilepsy, and her neurologist plans to start valproic acid for this type of epilepsy. She would like to know about the potential side effects of this medication. Which of the following is not a side effect of valproic acid?

a. Tremor

b. Hepatotoxicity

c. Hyperammonemia

d. Hyponatremia

e. Neural tube defects in children of mothers who take this medication

70. A 67-year-old man with a history of bipolar disorder and epilepsy is brought to the clinic by his wife. Over the past 6 months, since he was started on a new medication, he has become “meaner” and his wife states that she cannot stand him anymore, he yells all the time, and has become abusive and very aggressive. Which of the following medications is the patient most likely taking that would explain his behavior?

a. Lamotrigine

b. Valproic acid

c. Carbamazepine

d. Levetiracetam

e. Phenytoin

71. A 59-year-old man presents for evaluation of partial seizures and is prescribed an antiepileptic agent. About 2 weeks later he wakes up with a painful red eye and decreased visual acuity. He is diagnosed with acute closed-angle glaucoma. Which of the following is the most likely antiepileptic agent that he was prescribed recently?

a. Valproic acid

b. Levetiracetam

c. Phenytoin

d. Lamotrigine

e. Topiramate

72. A 42-year-old man with a history of hyperlipidemia and no prior history of epilepsy is brought to the emergency department after a seizure that witnesses described as generalized tonic–clonic, associated with tongue biting and urinary incontinence. The seizure lasted for less than a minute, and the patient was confused for about 20 minutes after the event. His neurologic examination is unremarkable. Which of the following statements is incorrect?

a. There is evidence in the literature to support the need for an EEG

b. Brain imaging is recommended

c. There are strong data to support the need to order routine blood count, glucose, and electrolytes

d. There is not enough evidence to support or refute the need for toxicology screen

e. An abnormal EEG may predict the risk of recurrence

73. Which of the following is correct regarding the EEG in Figure 5.7?

FIGURE 5.7 EEG (Courtesy of Dr. Joanna Fong)

a. There is poor reactivity of the posterior background

b. The patient has an occipital seizure

c. The posterior background shows δ-frequencies

d. An eye closure is recorded during this EEG page

e. The patient is sleeping

74. Which of the following is correct regarding EEG frequencies?

a. α-frequency is >13 Hz

b. β-frequency is 8 to 13 Hz

c. δ-frequency is <4 Hz

d. θ-frequency is 2 to 3 Hz

e. All of the above

75. Which of the following is not an activation procedure that is used during EEG recording to increase the diagnostic yield?

a. Hyperventilation

b. Sleep deprivation prior to the EEG

c. Noxious stimulation

d. Photic stimulation

e. Recording during sleep

76. Which of the following is the most likely diagnosis on the basis of the EEG shown in Figure 5.8?

FIGURE 5.8 EEG (Courtesy of Dr. Joanna Fong)

a. Hepatic encephalopathy

b. Generalized periodic pattern

c. A generalized seizure

d. Postcardiac arrest anoxia

e. Infantile spasms

77. Which of the following is consistent with the EEG shown in Figure 5.9?

a. Burst suppression

b. Triphasic waves

c. Hypsarrhythmia

d. Periodic lateralized epileptiform discharges (PLEDs)

e. A generalized seizure

78. Which of the following is consistent with the EEG shown in Figure 5.10?

a. Burst suppression

b. Triphasic waves

c. Generalized periodic pattern

d. Periodic lateralized epileptiform discharges (PLEDs)

e. Sharp waves

FIGURE 5.9 EEG (Courtesy of Dr. Joanna Fong)

FIGURE 5.10 EEG (Courtesy of Dr. Joanna Fong)

Questions 79–81

79. K complexes are seen predominantly in which stage of sleep?

a. Stage 1 sleep

b. Stage 2 sleep

c. Stage 3 sleep

d. Stage 4 sleep

e. REM sleep

80. How long after sleep onset does the first REM period normally occur?

a. 15 minutes

b. 30 minutes

c. 45 minutes

d. 60 minutes

e. 90 minutes

81. Which of the following is considered to be the circadian pacemaker?

a. Pineal gland

b. Suprachiasmatic nucleus in the anterior hypothalamus

c. Intralaminar thalamic nucleus

d. Suprachiasmatic nucleus in the posterior hypothalamus

e. Dorsolateral hypothalamus

82. Which of the following apnea–hypopnea indices (AHI) are diagnostic of moderate obstructive sleep apnea (OSA)?

a. 0 to 5/hour

b. 5 to 15/hour

c. 15 to 30/hour

d. 30 to 45/hour

e. 45 to 60/hour

83. A 43-year-old man presents with excessive daytime sleepiness. During a polysomnogram he has at least 9 episodes/hour, in which he stops breathing for approximately 10 seconds. Sometimes these are associated with arousals. Despite absent airflow he has respiratory effort during these episodes. Which of the following is correct?

a. This patient has central sleep apnea syndrome

b. Polysomnogram is not helpful for the diagnosis

c. This condition is associated with an increased risk of cardiovascular events

d. Alcohol and sedatives do not worsen this condition

e. Oxygen desaturation is rarely associated with these events

84. A 42-year-old obese woman presents with excessive daytime sleepiness. She has frequent arousals during the night and her husband reports that sometimes she stops breathing. A polysomnogram is performed and shows about 8 apneas/hour associated with desaturations. During these apneas in addition to no airflow there is no respiratory effort. Which of the following is correct regarding this condition?

a. This patient has partial or complete airway obstruction that results in hypopneas and apneas

b. There is a transient central cessation of respiratory drive

c. Some cases need surgical interventions to reduce the airway obstruction

d. This is the most common form of sleep disorder of breathing

e. All of the above

85. A 10-year-old boy has been reported to have episodes during which he wakes up screaming, hyperventilating, and is tachycardic and diaphoretic. He appears to be asleep during the episode, and when he awakens he is confused and does not recall the event. Which of the following is correct regarding his sleep disorder?

a. This is a REM parasomnia

b. This occurs out of sleep stage II

c. This is a nightmare

d. This occurs out of slow-wave sleep

e. This occurs more often during the last third of the night

86. A 16-year-old boy is reported to have episodes in which he walks around his house in the middle of the night. Once, during such an episode, he took the garbage out of his house and placed it in front of his neighbor’s door. His neighbor tried to talk to him, but he did not respond. When the neighbor shook him, the patient suddenly changed his behavior, started to scream, and became violent and confused. A sleep disorder was later diagnosed. Which of the following is correct?

a. This patient has a REM parasomnia

b. This patient has confusional arousals

c. This patient has narcolepsy

d. During these episodes, the EEG shows slow-wave sleep

e. This patient has sleep terrors

87. A 12-year-old boy wakes up in the middle of the night with frightening dreams of a big clown eating his parents. On awaking, he recalls the dream well and does not look confused. Which of the following is the most likely diagnosis?

a. Nightmares

b. Confusional arousals

c. Sleep terrors

d. REM sleep behavior disorder

e. Non-REM parasomnia

88. A 75-year-old man with a history of Parkinson’s disease is brought for evaluation by his wife who reports that he frequently has very vivid dreams, in which he screams, kicks, and punches. She is concerned because she has been punched at least twice. Which of the following is not correct?

a. This patient has a REM parasomnia

b. This condition can be seen in patients with multisystem atrophy

c. Polysomnogram demonstrates the absence of muscle tone during REM sleep

d. This patient has REM sleep behavior disorder (RBD)

e. This condition can be seen in patients with dementia with Lewy bodies

89. Which of the following statements regarding sleep phenomena is not correct?

a. Sleep paralysis is a feature of narcolepsy

b. Hypnagogic hallucinations occur on falling asleep

c. Hypnopompic hallucinations occur on waking up from sleep

d. Patients have recollection of nightmares

e. REM sleep behavior disorder (RBD) is associated with excessive atonia during REM sleep

90. A 14-year-old obese boy has recurrent episodes lasting up to 10 days, in which he sleeps almost continuously throughout the day and night, waking up only to eat and go to the bathroom. His mother reports that during these episodes he is in a bad mood. In between episodes he is a normal child. Which of the following is the most likely diagnosis?

a. Idiopathic hypersomnia

b. Narcolepsy

c. Kleine–Levine syndrome

d. Untreated obstructive sleep apnea syndrome

e. Delayed sleep phase syndrome

91. An 18-year-old woman is referred for evaluation of possible pseudoseizures. During the episodes she loses muscle tone and falls to the floor suddenly, being unable to move for about 40 to 50 seconds. These episodes are often preceded by emotional triggers, frequently laughter. On further questioning, she reports being conscious during these events. She also reports being very sleepy during daytime, having multiple episodes of falling asleep during the day, after which she feels refreshed. Which of the following is the most likely diagnosis?

a. Pseudoseizures

b. Gelastic seizures

c. Kleine–Levin syndrome

d. Narcolepsy with cataplexy

e. Idiopathic hypersomnia

92. Regarding narcolepsy with cataplexy, which of the following is incorrect?

a. CSF hypocretin levels are increased

b. During cataplexy, muscle tone is lost and patients are hyporeflexic or areflexic

c. Sleep paralysis is frequent

d. Mean sleep latency test (MSLT) needs to be performed to help make the diagnosis

e. γ-hydroxybutyrate is approved for the treatment of narcolepsy with cataplexy

93. A 16-year-old girl complains of excessive sleepiness during the day, with difficulty waking up to go to school in the morning. At night, she has difficulty falling asleep and tosses and turns for a long time. On the weekends, even though she usually goes out with her friends and sleeps late at night, she wakes up without problems late in the day and feels refreshed. Which of the following is the most likely diagnosis?

a. Psychophysiologic insomnia

b. Delayed sleep phase syndrome

c. Advanced sleep phase syndrome

d. Jet lag syndrome

e. Shift work sleep disorder

94. A 62-year-old man comes for evaluation of difficulty sleeping. He reports that he wakes up on his own every day at 4 AM, and cannot fall asleep again. He usually eats dinner at 5 PM and is very sleepy by 7 PM. Which of the following is the most likely diagnosis?

a. Psychophysiologic insomnia

b. Delayed sleep phase syndrome

c. Advanced sleep phase syndrome

d. Jet lag syndrome

e. Shift work sleep disorder

95. A 45-year-old female who works at the nursing unit desk comes for evaluation of sleepiness during the daytime and episodes of insomnia. She usually works between 11 PM and 7 AM 3 days/week, and between 7 AM and 3 PM2 days/week. During vacations, she tends to sleep well. Which of the following is the most likely diagnosis?

a. Psychophysiologic insomnia

b. Delayed sleep phase syndrome

c. Advanced sleep phase syndrome

d. Jet lag syndrome

e. Shift work sleep disorder

96. A 32-year-old woman comes for evaluation of excessive daytime sleepiness and insomnia. She reports that she cannot fall asleep well despite going to bed early every day and attempts to fall asleep by reading or watching television while in her bed. She says that as the night advances she gets more anxious and continuously thinks about falling asleep. Which of the following is the most likely diagnosis?

a. Psychophysiologic insomnia

b. Delayed sleep phase syndrome

c. Advanced sleep phase syndrome

d. Jet lag syndrome

e. Shift work sleep disorder

97. A 32-year-old woman complains of insomnia and excessive daytime sleepiness. She reports that when she goes to bed at night, she experiences an abnormal leg sensation that is hard to describe and is associated with an irresistible urge to move her legs. The urge to move her legs is temporarily relieved by moving them. Which of the following is the most likely diagnosis?

a. Restless legs syndrome (RLS)

b. Periodic limb movements of sleep (PLMS)

c. Psychophysiologic insomnia

d. Delayed sleep phase syndrome

e. Advanced sleep phase syndrome

98. Which of the following is incorrect regarding restless legs syndrome (RLS)?

a. Can be seen in patients with chronic renal failure

b. Can be associated with folate deficiency

c. Low ferritin levels are implicated in the pathophysiology of this condition

d. Low hypocretin levels are implicated in the pathophysiology of this condition

e. Dopamine agonists are used for the treatment of this condition

Answer Key

1. c

2. c

3. e

4. a

5. b

6. c

7. a

8. c

9. d

10. c

11. a

12. c

13. d

14. b

15. c

16. b

17. d

18. b

19. a

20. d

21. d

22. d

23. e

24. b

25. d

26. c

27. d

28. c

29. d

30. b

31. c

32. d

33. d

34. c

35. c

36. d

37. e

38. c

39. a

40. b

41. a

42. e

43. a

44. b

45. d

46. d

47. e

48. a

49. b

50. c

51. b

52. c

53. d

54. c

55. d

56. b

57. d

58. a

59. a

60. c

61. e

62. a

63. b

64. c

65. c

66. e

67. b

68. e

69. d

70. d

71. e

72. c

73. d

74. c

75. c

76. a

77. a

78. d

79. b

80. e

81. b

82. c

83. c

84. b

85. d

86. d

87. a

88. c

89. e

90. c

91. d

92. a

93. b

94. c

95. e

96. a

97. a

98. d

Answers

1. c

Lamotrigine is the correct choice. The prevalence of major congenital malformations in offspring of women with epilepsy ranges from 4% to 10%. This corresponds to a two- to fourfold increase from the expected prevalence in the general population. The currently available data suggest that this increased risk can be attributed, at least in part, to exposure to AEDs. However, it should be noted that increased risk of congenital malformations (compared to the general population) has been demonstrated in the offspring of women with epilepsy whose mothers were not taking any AEDs during pregnancy. Although robust long-term data are lacking, some of the newer AEDs, such as lamotrigine, appear to have a lower risk of teratogenesis and are felt to be safer than the other choices listed, all of which belong to the first (older) generation of AEDs. It is important to note that clearance of lamotrigine increases substantially during pregnancy, so the dose may have to be adjusted during this time and breakthrough seizures can occur.

Data regarding congenital malformations related to AED exposure have been building over the past few years in relation to commonly prescribed AEDs, such as carbamazepine, valproate, and lamotrigine. Interestingly, birth defect rates with carbamazepine monotherapy are lower than previously thought and some large studies report only marginally increased rates compared with different controls. Recent data also do not suggest adverse effects of carbamazepine on cognitive development. The prevalence of congenital malformations in association with lamotrigine appears to be similar to that of carbamazepine, and limited prospective studies on cognitive development do not indicate any adverse effects of lamotrigine. Malformation rates with valproate have consistently been two to three times higher compared with carbamazepine or lamotrigine. In addition, data suggest significant dose-dependent cognitive adverse events of in-utero use of valproate. Of the choices listed, valproic acid has the highest risk. It is important to remember that the greatest risk for cleft lip and palate, neural tube malformations, and congenital heart defects is during the first trimester. The neural tube closes during the critical period of weeks 3 and 4. Preconceptional folate supplementation is recommended to decrease the risk of neural tube defects (see Chapter 14). A daily dose of 0.4 mg/day of folate supplementation is recommended for all women of childbearing age, and a folate dose of up to 4–5 mg/day is recommended for all women with epilepsy taking AEDs.

Harden CL, Meador KJ, Pennell PB, et al. Practice parameter update: management issues for women with epilepsy. Focus on pregnancy (an evidence-based review): teratogenesis and perinatal outcomes. Report of the Quality Standards Subcommittee and Therapeutics and Technology Subcommittee of the American Academy of Neurology and American Epilepsy Society. Neurology. 2009; 73(2):126–132.

Harden CL, Pennell PB, Koppel BS, et al. Practice parameter update: management issues for women with epilepsy–focus on pregnancy (an evidence-based review): vitamin K, folic acid, blood levels, and breastfeeding: report of the Quality Standards Subcommittee and Therapeutics and Technology Assessment Subcommittee of the American Academy of Neurology and American Epilepsy Society. Neurology. 2009; 73:142–149.

Meador KJ, Baker GA, Browning N, et al. Cognitive function at 3 years of age after fetal exposure to antiepileptic drugs. N Engl J Med. 2009; 360(16):1597–1605.

Tomson T, Battino D. Teratogenic effects of antiepileptic medications. Neurolol Clin. 2009; 27:992–1002.

2. c

Topiramate is associated with weight loss. All of the other antiepileptics listed can cause weight gain. Additional side effects of topiramate that the patient should be educated about include drowsiness, word-finding difficulties, cognitive impairment, confusion, impaired memory, paresthesias, dizziness, nervousness, painful angle-closure glaucoma, and kidney stones.

Biton V. Weight change and antiepileptic drugs: health issues and criteria for appropriate selection of an antiepileptic agent. Neurologist. 2006; 12:163–167.

3. e

Temporal lobe epilepsy is often characterized by automatisms, altered consciousness, déjà vu phenomena, complex partial seizures, and olfactory hallucinations (discussed further in question 43). The fencer’s posture is associated with frontal lobe epilepsy and indicates epileptic activation of the supplemental motor area. It is described as external rotation and abduction of the contralateral arm from the shoulder, with head turning towards the same side of the arm posture. Frontal lobe seizures are discussed further in question 44.

O’Brien TJ, Mosewich RK, Britton JW, et al. History and seizure semiology in distinguishing frontal lobe seizures and temporal lobe seizures. Epilepsy Res. 2008; 82(2-3):177–182.

4. a

Gabapentin is neither an enzyme inducer nor inhibitor, so it has less potential interactions with other medications. It can be used as adjunctive therapy for partial seizures with or without secondary generalization. It is not used as monotherapy, given the availability of other more efficacious antiepileptic agents. Gabapentin can worsen generalized epilepsy, especially myoclonic epilepsy. The mechanism by which gabapentin exerts its anticonvulsant action is unknown. Its principal proposed mechanism of action, however, is through an interaction with the a2-δ subunit of presynaptic L-type voltage-regulated calcium channels. This subunit was recently identified as the specific binding site of gabapentin, as well as pregabalin, in the mammalian brain; binding of gabapentin and pregabalin may result in modulation of presynaptic neurotransmitter release.

Gabapentin is absorbed by an active transporter in the intestine. When the transporter becomes saturated, the absorption of gabapentin becomes nonlinear (i.e., a smaller percentage is absorbed at higher doses). Notably, this is in contrast to pregabalin, which has a linear absorption and, thus, has higher bioavailability. Gabapentin is renally excreted, and essentially no metabolism occurs before excretion. The most common side effects of gabapentin include fatigue, headache, nausea, dizziness, and ataxia. There are no significant drug interactions or idiosyncratic reactions. The other medications listed interact with the metabolism of various other drugs.

Díaz RA, Sancho J, Serratosa J. Antiepileptic drug interactions. Neurologist. 2008; 14:S55–S65.

5. b

The ketogenic diet has been reported to be effective in refractory cases of epilepsy in childhood, even when multiple antiepileptic trials have failed. It is typically initiated in the hospital by starvation for 1–2 days in order to induce ketosis. This is followed by a strict diet in which 80% to 90% of calories are derived from fat. Surgical procedures such as vagus nerve stimulation and even more invasive procedures such as corpus callosotomy should be used as the last resort in select cases. Resective surgical therapies do not play a significant role in the treatment of generalized epilepsies. Carbamazepine and phenytoin can frequently worsen generalized epilepsy and are unlikely to be useful.

Bough KJ, Rho JM. Anticonvulsant mechanisms of the ketogenic diet. Epilepsia. 2007; 48:43–58.

Hartman AL, Vining EP. Clinical aspects of the ketogenic diet. Epilepsia. 2007; 48:31–42.

6. c

Supportive care is the general recommendation for the management of a simple febrile seizure (FS). It is estimated that about 3% to 5% of children aged 5 months to 5 years have simple FS. Ninety percent of these events occur in the first 3 years of life. One-third of patients have at least one additional seizure. Risk factors for having a simple FS include family history of FS, prolonged neonatal ICU stay, developmental delay, and day care. Incidence does not increase in proportion to increase in temperature. No risk factors are found in 50% of children with an FS. The risk of afebrile epilepsy after FS is increased in children with developmental delay, abnormal neurologic examination, complex FS (defined below), and a family history of afebrile seizures. There is a <5% risk that patients with a simple FS will develop epilepsy. It is estimated that approximately 15% of patients with epilepsy have a history of FS.

Simple FS are characterized by the following: <15 minutes in duration, generalized seizure, lack of focality, neurologically normal examination, no persistent deficits, and negative family history for seizures. Complex FS occur in approximately 20% of FS and are characterized by the following: >15 minutes in duration, focal features, abnormal neurologic examination, seizure recurrence in <24 hours, postictal signs (Todd’s paralysis), and are more likely to be due to meningitis, encephalitis, or an underlying seizure disorder.

Prophylaxis is generally not needed, but can be considered for recurrent or prolonged seizures, afebrile seizures, after complex FS, and with an abnormal neurologic examination and developmental delay. Chronic prophylaxis commonly includes phenobarbital and valproic acid, whereas short-term prophylaxis could include diazepam and antipyretics, though definitive data on the use of antipyretics for the prevention of FS are lacking. Furthermore, the potential toxicities associated with available antiepileptic agents outweigh the relatively minor risks associated with simple FS. After reviewing the potential risks and benefits of available effective therapies for short- and long-term prophylaxis, the American Academy of Pediatrics concluded (in its clinical practice guideline on long-term management of children with FS) that neither continuous nor intermittent anticonvulsant therapy is recommended for children with one or more simple FS.

Berg AT, Shinnar S. Complex febrile seizures. Epilepsia. 1996; 37:126–133.

Knudsen FU. Febrile seizures: treatment and prognosis. Epilepsia. 2000; 41(1):2–9.

Steering Committee on Quality Improvement and Management, Subcommittee on Febrile Seizures American Academy of Pediatrics. Febrile seizures: clinical practice guideline for the long-term management of the child with simple febrile seizures. Pediatrics. 2008; 121:1281–1286.

7. a

Generalized epilepsy with febrile seizures plus (GEFS+) is considered to be a familial syndrome and SCN1D mutations are not a recognized cause. In contrast to febrile seizures (FS), which occur most commonly between 6 months and 5 years of age, the phenotype of “febrile seizures plus” includes patients in whom FS continue past the defined upper limit of age. GEFS+ may also be associated with afebrile generalized tonic-clonic (GTC) seizures. One-third of patients have other seizure types as well. The pattern of inheritance is usually complex, although initial genetic discoveries first identified an autosomal dominant familial pattern. Mutations of a number of ion channel genes have been identified in GEFS+ kindreds. These include sodium channel (SCN) subunits (SCN1A, SCN1B, and SCN2A) and GABAA receptor subunit genes (GABRD and GABRG2). The result is increased sodium channel activity or impaired GABA activity, ultimately leading to increased cortical hyperexcitability. The most frequently reported mutation is SCN1A, which encodes the pore-forming α-subunit of the sodium channel and comprises four transmembrane domains. The EEG usually shows generalized spike–wave or polyspikes.

Scheffer IE, Zhang YH, Jansen FE, et al. Dravet syndrome or genetic (generalized) epilepsy with febrile seizures plus? Brain Dev. 2009; 31(5):394–400.

Zucca C, Redaelli F, Epifanio R, et al. Cryptogenic epileptic syndromes related to SCN1A: twelve novel mutations identified. Arch Neurol. 2008; 65(4):489–494.

8. c

Rasmussen’s syndrome is a rare, but severe, inflammatory brain disorder characterized by progressive unilateral hemispheric atrophy, associated progressive neurologic dysfunction (hemiparesis and cognitive deterioration), and intractable focal seizures (epilepsia partialis continua). Imaging reveals slowly progressive development of focal cortical atrophy, which correlates to the clinical findings. It has been postulated that antibodies to glutamate receptor-3 (GLUR3) may play a pathogenic role, although the available data are conflicting and the specificity of GLUR3 antibodies in the pathogenesis of Rasmussen’s encephalitis has been challenged in recent years. The focal cortical atrophy is progressive and eventually spreads to the surrounding cortical areas in the same hemisphere, and thus, the best treatment option for the patient’s intractable seizures is the surgical approach with hemispherectomy. The other listed options are all developmental cortical malformations, with porencephaly often resulting from an ischemic insult in utero.

Bien CG, Granata T, Antozzi C, et al. Pathogenesis, diagnosis and treatment of Rasmussen encephalitis: a European consensus statement. Brain. 2005; 128:454–471.

Bien CG, Urbach H, Deckert M, et al. Diagnosis and staging of Rasmussen’s encephalitis by serial MRI and histopathology. Neurology. 2003; 58:250–257.

9. d

Most progressive myoclonic epilepsies are due to either lysosomal storage disorders and/or mitochondrial disorders. They are characterized by progressive cognitive decline, myoclonus (epileptic and nonepileptic), seizures (tonic–clonic, tonic, and myoclonic), and may be associated with ataxia or movement disorders. Examples include Lafora body disease, Unverricht Lundborg syndrome, neuronal ceroid lipofuscinosis, myoclonic epilepsy with ragged red fibers (MERRF), and sialidosis. Valproic acid is often the first-line treatment for myoclonic epilepsy. Caution is advised with use of valproic acid in patients with mitochondrial mutations, such as POLG gene mutations, because fulminant hepatic failure may result. Other treatments include clonazepam, levetiracetam, topiramate, and zonisamide. Lamotrigine is sometimes used, but caution is advised because it rarely may worsen myoclonic seizures. Gabapentin, carbamazepine, pregabalin, and vigabatrin are also known to exacerbate some myoclonic epilepsies.

Genton P, Gelisse P. Antimyoclonic effect of levetiracetam. Epileptic Disord. 2000; 2:209–212.

Malphrus AD, Wilfong AA. Use of the newer antiepileptic drugs in pediatric epilepsies. Curr Treat Option Neurol. 2007; 9(4):256–267.

10. c

Fosphenytoin is an IV prodrug of phenytoin. It is composed of a disodium phosphate ester that is water soluble and less alkaline than phenytoin. It does not include propylene glycol and ethyl alcohol as a solvent vehicle as is the case with IV phenytoin. Fosphenytoin can be loaded at a faster rate, but because the fosphenytoin needs to be converted into phenytoin in plasma, the rate of rise of serum levels is approximately equal to that of phenytoin. Compared to phenytoin, fosphenytoin is not associated with purple glove syndrome, it can be given more rapidly intravenously, its administration is associated with a lower occurrence of cardiovascular side effects, such as hypotension, and it can be given intramuscularly. Purple glove syndrome may ensue when phenytoin infiltrates into the subcutaneous tissue, resulting in swelling, pain, and discoloration of the extremity because of blood vessel leakage. The most common side effects of IV fosphenytoin include pruritus, as well as the other less problematic and typical phenytoin side effects, such as dizziness, nystagmus, and drowsiness.

Bradley WG, Daroff RB, Fenichel GM, et al. Neurology in Clinical Practice, 5th ed. Philadelphia, PA: Elsevier; 2008.

Fischer JH, Patel TV, Fischer PA. Fosphenytoin: clinical pharmacokinetics and comparative advantages in the acute treatment of seizures. Clin Pharmacokinet. 2003; 42(1):33–58.

11. a

Topiramate is typically not associated with myoclonic seizure exacerbation. Lamotrigine, gabapentin, carbamazepine, pregabalin, and vigabatrin are known to exacerbate some myoclonic epilepsies.

Guerrini R, Belmonte A, Parmeggiani L, et al. Myoclonic status epilepticus following high-dosage lamotrigine therapy. Brain Dev. 1999; 21:420–424.

Welty TE. Juvenile myoclonic epilepsy: epidemiology, pathophysiology, and management. Paediatr Drugs. 2006; 8(5):303–310.

12. c

Valproic acid is the only antiepileptic medication listed that is a hepatic enzyme inhibitor. All others listed are hepatic enzyme inducers. This information is important to know in order to safely prescribe a concurrent antiepileptic or other medications. Concurrent use of valproic acid with medications that undergo the same hepatic enzyme metabolism, may result in dangerously elevated serum levels of these medications because their metabolism is inhibited. An example would be concurrent valproic acid and warfarin, which could result in elevated INR levels and, thus, increased bleeding risk.

Wyllie E, Gupta A, Lachhwani DK (Eds). The Treatment of Epilepsy: Principles and Practice, 4th ed. Philadelphia, PA: Lippincott Williams & Wilkins; 2005.

13. d

-Hz spike and wave is characteristic for absence epilepsy. The other options are benign EEG patterns unassociated with seizures (also known as normal variants). Absence epilepsy has a peak age around 6 years and more often affects girls (70%). These patients are generally normal neurologically. Absence epilepsy is characterized by multiple daily spells lasting a few seconds. They begin and end abruptly and interrupt whatever activity is being carried out. During a seizure, there will often be a blank stare; automatisms such as lip smacking, nose rubbing, and picking at clothes may also be present, especially with longer episodes. These seizures are classically provoked by hypoglycemia and hyperventilation. Mild ictal jerks of eyelids, eyes, and eyebrows may occur at the onset of the seizure. The thalamus is implicated in the generation and sustainment of absence epilepsy with the low-threshold (T-type) calcium channels of thalamic neurons playing a central role in thalamocortical interactions. First-line treatment includes ethosuximide (which acts via T-type calcium channel inhibition). Valproic acid, lamotrigine, topiramate, and zonisamide are also used. Notably, the use of lamotrigine has been associated with aggravation of absence seizures on rare occasions. In a double-blind, randomized, controlled clinical trial, the efficacy, tolerability, and neuropsychologic effects of ethosuximide, valproic acid, and lamotrigine were compared in children with newly diagnosed childhood absence epilepsy. Ethosuximide and valproic acid were found to be more effective than lamotrigine. Notably, ethosuximide was also associated with fewer adverse attentional effects, and therefore, it is considered to be the best choice for initial empirical monotherapy in children with absence epilepsy.

Valproic acid or lamotrigine are often the drugs of choice when there are concurrent generalized tonic-clonic (GTC) and absence seizures. It is important to note that GABAB receptors promote activation of T-type calcium channels. Therefore, some GABAergic drugs can exacerbate absence seizures.

Antiepileptic medications can often be discontinued as the child grows older, if the EEG is normal, and there have been no seizures for 1 to 2 years. Absence epilepsy carries a good prognosis. Eighty percent of children have remission through adolescence and at least 90% eventually have remission overall.

Glauser TA, Cnaan A, Shinnar S, et al. Ethosuximide, valproic acid, and lamotrigine in childhood absence epilepsy. N Engl J Med. 2010; 362(9):790–799.

Wyllie E, Gupta A, Lachhwani DK (Eds). The Treatment of Epilepsy: Principles and Practice, 4th ed. Philadelphia, PA: Lippincott Williams & Wilkins; 2005.

14. b

The adult pattern of normal posterior dominant α-rhythm in older children and adults is usually seen by the age of 8 to 10 years. The following is a review of adult EEG frequencies:

–β >13 Hz
–α 8 to 13 Hz
–θ 4 to 7 Hz
–δ <4 Hz

Bradley WG, Daroff RB, Fenichel GM, et al. Neurology in Clinical Practice, 5th ed. Philadelphia, PA: Elsevier; 2008.

Levin K, Luders H. Comprehensive Clinical Neurophysiology, Philadelphia, PA: WB Saunders Company; 2000.

15. c

Periodic lateralized epileptiform discharges (PLEDs) would be expected, and are also discussed in question 78. PLEDs consist of unilateral or bilateral, independent, high-amplitude, sharp, and slow-wave complexes at 0.5 to 3 Hz. Any destructive process such as anoxia, HSV encephalitis, stroke, and tumor can cause PLEDs. Triphasic waves are seen with hepatic coma, anoxia, drug toxicity, and other toxic and metabolic encephalopathies. Triphasic waves are generalized and maximal bifrontal, and consist of a prominent positive wave preceded and followed by minor negative waves at 0.5- to 2-Hz intervals. Wicket spikes belong to the benign normal variants. Polyspikes and fast spike–wave complexes are true epileptiform discharges.

Levin K, Luders H. Comprehensive Clinical Neurophysiology. Philadelphia, PA: WB Saunders Company; 2000.

16. b

Phenytoin, carbamazepine, gabapentin, and lamotrigine have all been associated with aggravation of absence seizures and even absence status epilepticus in children with absence epilepsy. Topiramate does not have this association.

Kaplan PW, Drislane FW (Eds). Nonconvulsive Status Epilepticus. New York: Demos Medical Publishing; 2009.

Manning JP, Richards DA, Bowery NG. Pharmacology of absence epilepsy. Trends Pharmacol Sci. 2003; 24(10):542–549.

Shorvon S, Walker M. Status epilepticus in idiopathic generalized epilepsy. Epilepsia. 2005; 46(Suppl 9):73–79.

17. d

Many enzyme-inducing antiepileptics (phenytoin, carbamazepine, phenobarbital, oxcarbazepine, and topiramate at doses >200 mg/day) increase metabolism of oral contraceptives. Antiepileptic medications with minimal oral contraceptive interaction include valproic acid, gabapentin, pregabalin, levetiracetam, zonisamide, tiagabine, and topiramate (at doses <200 mg/day).

Crawford P. Best practice guidelines for the management of women with epilepsy. Epilepsia 2005; 46(Suppl 9):117–124.

18. b

This EEG reveals a run of 3-Hz spike and wave discharges typically seen with absence seizures in childhood absence epilepsy. A paroxysmal 3-Hz spike and wave pattern emerges abruptly out of a normal background and suddenly ceases after a few seconds. Absence epilepsy is discussed further in question 13.

Levin K, Luders H. Comprehensive Clinical Neurophysiology, Philadelphia, PA: WB Saunders Company; 2000.

19. a, 20. d

The EEG in Figure 5.2 reveals polyspikes in a patient with juvenile myoclonic epilepsy (JME), and the history is also very suggestive of this disorder. Valproic acid is typically the first-line agent for JME. JME is one of the idiopathic generalized epilepsies. Onset is typically between 8 and 24 years (peaks in teens). Development is typically normal. Boys and girls seem to be equally affected. Myoclonic seizures constitute the most frequent seizure type. These are usually described as large-amplitude and bilateral simultaneous myoclonic jerks. Myoclonic seizures are predominantly seen on awakening, and the patient often complains about being “clumsy” in the morning and frequently dropping things. Falls are not infrequent. There is typically no loss of consciousness, although myoclonic seizures can occasionally be followed by a generalized tonic-clonic (GTC) seizure. Most patients have infrequent GTC seizures, which usually also occur on awakening. Some patients with JME also have typical absence seizures. The EEG reveals generalized 4- to 6-Hz polyspike and wave discharges interictally. Ictally, trains of spikes are seen, which are commonly triggered by photic stimulation (during EEG recordings). The first-line treatment is with valproic acid. Second-line treatments include lamotrigine, levetiracetam, topiramate, and zonisamide. Carbamazepine and phenytoin should be avoided because they may lead to worsening of myoclonic seizures, similar to the worsening of childhood absence epilepsy seen with these agents. Good control will generally require lifelong treatment and avoidance of triggers, such as alcohol intake and lack of sleep.

Auvin S. Treatment of juvenile myoclonic epilepsy. CNS Neurosci Ther. 2008; 14(3):227–233.

Levin K, Luders H. Comprehensive Clinical Neurophysiology, Philadelphia, PA: WB Saunders Company; 2000.

Ropper AH, Samuels MA. Adams and Victor’s principles of Neurology, 9th ed. New York: McGraw-Hill; 2009.

21. d, 22. d

The history and EEG in Figure 5.3 suggest benign childhood epilepsy with centrotemporal spikes (benign rolandic epilepsy of childhood). The EEG classically reveals bilateral independent centrotemporal spikes on a normal background. The discharges on the two sides can be either independent or synchronized. They may extend beyond the centrotemporal regions. Although the spikes on the EEG appear in the centrotemporal area, the temporal lobe is not the generator of these spikes. Rather, they are felt to be generated in the base of the rolandic fissure.

Benign childhood epilepsy with centrotemporal spikes (benign rolandic epilepsy of childhood) is fairly common and accounts for about 25% of childhood seizures. Onset is usually between 2 and 13 years of age, and the condition typically resolves in the midteenage years. Seizures are characterized by focal motor, sensory, or autonomic manifestations involving predominantly the face, mouth, throat, or extremities, although secondary generalization can occur. These are seizures that classically occur nocturnally (70% only in sleep, 15% only awake, and 15% both). EEG is characterized by the presence of independent bilateral, repetitive, broad, centrotemporal interictal EEG spikes on a normal background. The discharges are thought to arise from the vicinity of the precentral and postcentral gyri in the lower suprasylvian region. The characteristic EEG spike pattern is inherited as an autosomal dominant trait with variable penetrance. Normal development, physical examination, and brain imaging is the rule, though there are exceptions. Seizures respond well to certain antiepileptic medication and carbamazepine is usually considered the first line of therapy in the United States. It is important to note that it is often not necessary to treat with AEDs unless seizures are prolonged or frequent; some advocate waiting for two or more seizures to occur before initiating treatment. If antiepileptic medications are started, they can generally be stopped after adolescence. (Only 10% continue to have seizures 5 years after onset.)

Bouma PA, Bovenkerk AC, Westendorp RG, et al. The course of benign partial epilepsy of childhood with centrotemporal spikes: a meta-analysis. Neurology. 1997; 48(2):430–437.

Heijbel J, Blom S, Rasmuson M. Benign epilepsy of childhood with centrotemporal EEG foci: a genetic study. Epilepsia. 1975; 16:285–293.

Levin K, Luders H. Comprehensive Clinical Neurophysiology, Philadelphia, PA: WB Saunders Company; 2000.

23. e

The EEG in Figure 5.4 reveals periodic lateralized epileptiform discharges (PLEDs). PLEDs occur in acute lateralized pathology, such as a stroke, HSV encephalitis, rapidly expanding tumor, or any other destructive process to the brain parenchyma. PLEDs are discussed further in question 15.

Levin K, Luders H. Comprehensive Clinical Neurophysiology, Philadelphia, PA: WB Saunders Company; 2000.

24. b,

25. d

The EEG in Figure 5.5 reveals hypsarrhythmia, which is the most common interictal EEG correlate of infantile spasms. Hypsarrhythmia is characterized by abnormal interictal high-amplitude slow waves on a background of irregular multifocal spikes. These waves and spikes have no consistent pattern or rhythm and vary in duration and size, resulting in a chaotic-appearing EEG record. Hypsarrhythmia disappears ictally during a cluster of spasms and/or REM sleep.

Infantile spasms occur during the first year of life (typically 3–8 months), and are discussed further in question 45. They are characterized by sudden tonic extension or flexion of limbs and axial body, often occurring in clusters, and especially shortly after awakening. West syndrome is a triad of infantile spasms, hypsarrhythmia, and mental retardation. This disorder often occurs due to pre/peri/postnatal insults, tuberous sclerosis, cerebral dysgenesis, and others. Treatment with ACTH is generally first line, but it is expensive, especially in the United States. Other treatments include corticosteroids, vigabatrin, clonazepam, levetiracetam, topiramate, pyridoxine, and valproic acid. Vigabatrin has been associated with retinal toxicity.

Levin K, Luders H. Comprehensive Clinical Neurophysiology. Philadelphia, PA: WB Saunders Company; 2000.

Mackay MT, Weiss SK, Adams-Webber T, et al. Practice parameter: medical treatment of infantile spasms: report of the American Academy of Neurology and the Child Neurology Society. Neurology. 2004; 62:1668–1681.

Ropper AH, Brown RH. Adams and Victor’s Principles of Neurology, 8th ed. New York: McGraw-Hill; 2005.

Wong M, Trevathan E. Infantile Spasms. Pediatr Neurol. 2001; 24:89–98.

26. c, 27. d, 28. c

Phenytoin is used for the treatment of partial and/or generalized tonic-clonic (GTC) seizures (primary or secondary). Its primary mechanism of action is inhibition of voltage-dependent neuronal sodium channels. It undergoes predominantly liver metabolism, although there is also minimal renal metabolism. Patients who are in low-protein disease states (such as liver failure, etc.) need to be followed with free phenytoin levels because of less available protein for binding, making the total levels unreliable. It is important to understand that phenytoin exhibits nonlinear (zero-order) kinetics, as the metabolic pathways responsible for its metabolism become saturated. This means that when the dose of phenytoin is increased beyond a certain point, its plasma concentration at steady state will no longer increase in a proportionate manner, rather small dose changes may result in a large/toxic increment in plasma concentrations. In general, phenytoin approaches zero-order kinetics at total levels >10 to 15 μg/mL and small dose increments can potentially cause large increases in the serum level.

Idiosyncratic reactions caused by phenytoin include aplastic anemia, Stevens–Johnson syndrome, and hepatic failure. Other side effects include thrombocytopenia, lymphadenopathy, gingival hyperplasia, acne, coarse facial features (also called “phenytoin facies,” from hypertrophy of subcutaneous facial tissue), hirsutism, purple glove syndrome (with intravenous administration), nystagmus, ataxia, dysarthria, diplopia, nausea, dizziness, and drowsiness. Phenytoin can also cause folate deficiency and increased vitamin D metabolism, resulting in premature osteoporosis. Chronically, its use has been associated with a usually mild peripheral neuropathy and with cerebellar, but not cortical atrophy. Acutely, the IV form can cause phlebitis, pain, burning, hypotension, and cardiac conduction abnormalities. Phenytoin is a liver enzyme inducer, so it can increase metabolism of many other drugs.

There are variations of calculating loading and correcting doses of phenytoin. A general simple formula for calculating a supplementing (or loading) IV bolus of phenytoin is as follows: (target total phenytoin level – current total phenytoin level) × (kilogram body weight × volume of distribution). The therapeutic range for phenytoin is 10 to 20 μg/mL. The range for volume of distribution for phenytoin is 0.5 to 1 L/kg, with an average of 0.8 L/kg often used. If we insert the numbers from the case into the formula, the calculation will be as follows: (15 – 10) × (75 × 0.8) = 300 mg. An accurate reassessment of new levels can be obtained by checking free and total levels approximately 2 hours after the IV load.

Ahn JE, Cloyd JC, Brundage RC, et al. Phenytoin half-life and clearance during maintenance therapy in adults and elderly patients with epilepsy. Neurology. 2008; 71(1):38–43.

Ropper AH, Samuels MA. Adams and Victor’s Principles of Neurology, 9th ed. New York: McGraw-Hill; 2009.

Schachter SC. Review of the mechanisms of action of antiepileptic drugs. CNS Drugs. 1995; 4:469–477.

29. d, 30. b, 31. c

The correcting IV bolus for valproic acid is as follows: (target total valproic acid level – current total valproic acid level) × (kilogram body weight × volume of distribution). The therapeutic range for valproic acid is 50 to 100 μg/mL. The range for volume of distribution for valproic acid is 0.1 to 0.3 L/kg, with an average of 0.2 L/kg often used. If we apply this formula to the case, the result is as follows: (100 – 70) × (70 × 0.2) = 420 mg.

Valproic acid has broad-spectrum antiseizure activity and is commonly used in partial, generalized tonic-clonic (GTC), absence, myoclonic, and tonic seizures, as well as infantile spasms. Its mechanism of action is by sodium and T-type calcium channel antagonism, and it also works as an agonist at the GABAA receptor. It primarily undergoes liver metabolism and is a hepatic enzyme inhibitor. Side effects include cognitive and gastrointestinal complaints. Infrequently, it can cause increased liver enzymes and, rarely, idiosyncratic fatal hepatitis (most common in those <2 years of age). Chronically, it can cause weight gain, hair thinning, polycystic ovarian syndrome, acne, menstrual irregularities, tremor, pancreatitis, and thrombocytopenia. Cerebellar atrophy occurs with long-term phenytoin use, but not with valproic acid.

Valproic acid significantly increases the half-life of lamotrigine by 24 to 48 hours. Initiation of as little as 500 mg of valproic acid in chronic lamotrigine users may necessitate an immediate 50% reduction in the dose of lamotrigine.

Löscher W. Basic pharmacology of valproate: a review after 35 years of clinical use for the treatment of epilepsy. CNS Drugs. 2002; 16:669–694.

Ropper AH, Samuels MA. Adams and Victor’s Principles of Neurology, 9th ed. New York: McGraw-Hill; 2009.

32. d, 33. d, 34. c, 35. c

Carbamazepine is used for partial or secondarily generalized tonic-clonic (GTC) seizures, although it is important to remember that it can rarely worsen some generalized epilepsies (including myoclonic and absence epilepsies), similar to phenytoin. Its primary mode of action is via blockade of sodium channels, which leads to a decrease/prevention of repetitive firing in depolarized neurons. Side effects include dizziness, vertigo, fatigue, drowsiness, diplopia, nystagmus, headache, nausea, vomiting, elevated liver function tests, hyponatremia, and ataxia. Serious idiosyncratic reactions include Stevens–Johnson syndrome, leukopenia, and aplastic anemia.

Carbamazepine undergoes liver metabolism with renal excretion of metabolites, so caution is advised with kidney or liver failure. Carbamazepine is also a hepatic enzyme inducer, and undergoes autoinduction. The dose must be titrated up gradually to allow tolerance to develop to its CNS side effects, but also to avoid early toxicity as carbamazepine “autoinduces” the hepatic enzymes responsible for its own metabolism. If carbamazepine is started at too high of a dose, or titrated too fast, the result would be elevated carbamazepine levels with accompanying toxicity early on, as the hepatic enzymes responsible for carbamazepine’s metabolism have not been fully activated (autoinduced) yet. It is therefore important to remember that carbamazepine’s half-life decreases from 30 hours to 10 to 20 hours after the first few days to weeks of use. Autoinduction is completed after 3 to 5 weeks of a fixed dosing regimen. Plasma concentrations decrease in the first 1 to 2 months, and during this time, the dose of carbamazepine should be gradually increased. Therefore, carbamazepine would not be a good option if quick control of new-onset, frequent seizures was desired. Of note, oxcarbazepine does not undergo autoinduction and can be titrated faster.

Oxcarbazepine is a structural derivative of carbamazepine, and is reduced to 10-monohydroxy-carbamazepine and unlike carbamazepine does not undergo oxidation to epoxide. Carbamazepine on the other hand is oxidized to 10,11-carbamazepine epoxide, which is the principal metabolite of carbamazepine. It is important to remember that the 10,11-carbamazepine epoxide is pharmacologically active and responsible for many of the side effects seen with carbamazepine use. Because of these differences, oxcarbazepine has less side effects, overall, as compared to carbamazepine. Oxcarbazepine has less liver enzyme induction, no autoinduction (and can thus be titrated more rapidly), and is used for the same seizure types as carbamazepine, having the same mechanism of action, metabolic pathways, and side-effect profile. Approximately 30% of patients who have a history of a rash with carbamazepine will also develop a rash when exposed to oxcarbazepine.

Valproic acid inhibits the metabolism of the pharmacologically active 10,11-carbamazepine epoxide (the principal metabolite of carbamazepine). Thus, although the carbamazepine level may be normal, the patient may experience toxicity because of elevated 10,11-carbamazepine epoxide levels. The 10,11-carbamazepine epoxide is not routinely measured, but can be ordered specifically if there are concerns about toxicity.

Koch MW, Polman SK. Oxcarbazepine versus carbamazepine monotherapy for partial onset seizures. Cochrane Database Syst Rev. 2009;CD006453.

Porter RJ. How to initiate and maintain carbamazepine therapy in children and adults. Epilepsia. 1987; 28(Suppl 3):S59–S63.

Purcell TB, McPheeters RA, Feil M, et al. Rapid oral loading of carbamazepine in the emergency department. Ann Emerg Med. 2007; 50(2):121–126.

Tudur SM, Marson AG, Clough HE, et al. Carbamazepine versus phenytoin monotherapy for epilepsy. Cochrane Database Syst Rev. 2002; (2):CD001911.

36. d

Benzodiazepines are broad-spectrum antiepileptic medications used most commonly for partial, generalized tonic-clonic (GTC), absence, and myoclonic seizures, as well as status epilepticus. They work as GABAA agonists. Binding to the GABAA receptor leads to subsequent activation of chloride channels and, as a result, hyperpolarization of the neuronal membrane and decreased neuronal excitability. Benzodiazepines, in general, undergo liver metabolism and renal excretion of their metabolites.

Brunton LL, Lazo JS, Parker KL, eds. Goodman and Gilmans’ Pharmacological Basis of Therapeutics, 11th ed. New York: McGraw-Hill; 2005.

37. e, 38. c

Lamotrigine is typically well tolerated as long as it is introduced gradually with a slow titration, although dizziness, blurred vision, diplopia, and ataxia may be seen. Stevens–Johnson syndrome, or toxic epidermal necrolysis, is associated with concurrent use of valproic acid or rapid lamotrigine titration, and this risk appears to be increased in ages younger than 16 years. Cognitive disturbances are typically not seen with lamotrigine. Lamotrigine is a broad-spectrum antiepileptic medication and is used for partial and generalized tonic-clonic seizures (GTC), as well as generalized seizures of Lennox–Gastaut syndrome. It has also been used for absence and myoclonic seizures, although it is not the first line of therapy for these types of seizures. It works as a sodium channel antagonist and also inhibits glutamate release. It undergoes liver metabolism with renal excretion of metabolites.

Oral contraceptives and hormone replacement therapy increase lamotrigine clearance and, thus, decrease serum lamotrigine levels. This effect appears to be limited to contraceptives containing ethinylestradiol. Progesterone-only medications do not appear to have this effect. During pregnancy, lamotrigine clearance may increase up to 65%, which may result in breakthrough seizures. Therefore, monitoring of lamotrigine serum levels with dose adjustments is recommended during pregnancy and after delivery.

de Haan GJ, Edelbroek P, Segers J, et al. Gestation-induced changes in lamotrigine pharmacokinetics: a monotherapy study. Neurology. 2004; 63:571–573.

LaRoche SM, Helmers SL. The new antiepileptic drugs: scientific review. JAMA. 2004; 291:605–614.

Tran TA, Leppik IE, Blesi K, et al. Lamotrigine clearance during pregnancy. Neurology 2002; 59:251–255.

39. a, 40. b

Topiramate is a broad-spectrum antiepileptic used for partial, generalized tonic-clonic (GTC) and absence seizures, and for Lennox–Gastaut syndrome. It has multiple mechanisms of action, including voltage-dependent sodium channel antagonism, enhancement of GABA activity through a nonbenzodiazepine site on GABAA receptors, and antagonism of AMPA/kainate glutamate receptors. It is predominantly excreted unchanged in urine with minimal liver metabolism.

Similar to zonisamide, topiramate is also a weak carbonic anhydrase inhibitor, which explains the potential risk of renal stone formation in patients treated with these agents. Other side effects include paresthesias, decreased appetite, weight loss, dizziness, fatigue, and cognitive complaints, such as word-finding difficulty and slowed thinking. Acute angle-closure glaucoma has been reported, but is rare.

Glauser TA, Dlugos DJ, Dodson WE, et al. Topiramate monotherapy in newly diagnosed epilepsy in children and adolescents. J Child Neurol. 2007; 22:693–699.

Meldrum BS. Update on the mechanism of action of antiepileptic drugs. Epilepsia. 1996; 37(Suppl 6):S4–S11.

41. a

Lacosamide works by selective enhancement of slow inactivation of voltage-dependent sodium channels. The result is inhibition of repetitive neuronal firing and stabilization of hyperexcitable neuronal membranes. Lacosamide is also known to interfere with the activity of the collapsing response mediator protein-2 (CRMP-2), a cell protein involved in neuronal differentiation and axonal guidance. The nature of the interaction between lacosamide and CRMP-2 and its role in seizure control are unclear. Lacosamide is FDA approved as an adjunct for partial-onset seizures in patients aged 17 years and older. It is available in oral or IV formulation. It is eliminated primarily by renal excretion and has little drug–drug interaction with other antiepileptic medications. Dizziness and nausea are the most common side effects.

Beyreuther BK, Freitag J, Heers C, et al. Lacosamide: a review of preclinical properties. CNS Drug Rev. 2007; 13:21–42.

Perucca E, Yasothan U, Clincke G, et al. Lacosamide. Nat Rev Drug Discov. 2008; 7:973–974.

42. e

Rufinamide is not metabolized by the cytochrome P (CYP) 450 system. Rufinamide modulates the activity of neuronal sodium channels, resulting in prolongation of the inactive state of the channel. It is FDA approved as an adjunctive treatment of seizures associated with Lennox–Gastaut syndrome in children 4 years and older, and adults.

Rufinamide undergoes extensive metabolism, with only 4% excreted as parent drug. Rufinamide is primarily metabolized via enzymatic hydrolysis of the carboxylamide group to form carboxylic acid. This metabolic route is not CYP 450 dependent. There are no known active metabolites. Elimination of rufinamide is predominantly via urine. Plasma half-life of rufinamide is approximately 6 to 10 hours. Rufinamide shows little or no inhibition of most CYP 450 enzymes at clinically relevant concentrations, with weak inhibition of CYP 2E1. Rufinamide is a weak inducer of the CYP 3A4 enzyme.

Perucca E, Cloyd J, Critchley D, et al. Rufinamide: clinical pharmacokinetics and concentration-response relationships in patients with epilepsy. Epilepsia. 2008; 49:1123–1141.

43. a

This patient has complex partial seizures originating from the temporal lobe, most likely from the mesial temporal lobe region. These are often associated with mesial temporal lobe sclerosis. Complex partial seizures arise from a focal area of epileptogenicity, and unlike simple partial seizures, are associated with impairment of consciousness.

Mesial temporal lobe seizures are characterized by behavioral arrest, and may be preceded by an aura (a simple partial seizure), such as a rising epigastric sensation, nausea, olfactory and/or gustatory hallucinations, a sensation of fear and terror, or other emotional changes, as well as autonomic manifestations such as tachycardia, respiratory changes, face flushing, or pallor. Patients may also experience dysmnesic manifestations such as déjà vu (sensation of familiarity as if an experience has occurred before, although it has not), déjà entendu (if the experience is auditory), jamais vu (sensation that a familiar experience is new, although it is not), jamais entendu (if the latter experience is auditory), or panoramic vision (a rapid recollection of episodes from the past). During the seizure, the patient may also have automatisms, which are involuntary complex motor activities, such as nose picking, lip smacking, chewing, and picking with the hands. Typically patients have postictal confusion, which is not present in absence seizures.

Frontal lobe seizures are abrupt in onset, brief, and predominantly associated with elementary motor manifestations, but may include complex automatisms. They frequently occur in sleep, often in clusters. Parietal lobe seizures are predominantly associated with episodic sensory symptoms, although clinical localization may be difficult as parietal discharges propagate to other brain regions. Occipital lobe seizures usually present with visual phenomena. Given the focal discharges on the EEG and characteristic clinical features, this patient does not have absence seizures.

Bradley WG, Daroff RB, Fenichel GM, et al. Neurology in Clinical Practice, 5th ed. Philadelphia, PA: Elsevier; 2008.

Browne TR, Holmes GL. Handbook of Epilepsy. 4th ed. Philadelphia, PA: Lippincott Williams & Wilkins; 2008.

44. b

This patient has seizures coming from his frontal lobe, more specifically the right supplementary motor area (SMA). The typical semiology of these seizures has been referred to as “fencer’s posture,” a tonic posture in which the patient exhibits deviation of the eyes and head, as well as tonic arm extension to the side contralateral to the hemisphere where seizures are originating. These seizures are frequent, occurring in clusters or many times per day, and frequently arising during sleep. They are usually difficult to treat with medications.

Given the lateralization in this case, the left side is unlikely to be the origin of these seizures. Temporal lobe seizures are described in question 43. Parietal seizures are predominantly associated with episodic sensory symptoms, which include positive symptoms such as tingling or, less commonly, negative symptoms such as asomatognosia. In posterior parietal seizures, visual phenomena may occur.

Bradley WG, Daroff RB, Fenichel GM, et al. Neurology in Clinical Practice, 5th ed. Philadelphia, PA: Elsevier; 2008.

Browne TR, Holmes GL. Handbook of Epilepsy, 4th ed. Philadelphia, PA: Lippincott Williams & Wilkins; 2008.

45. d

This patient has infantile spasms with an EEG (Figure 5.5) that shows hypsarrhythmia. These patients usually have developmental arrest and a poor prognosis with respect to neurologic recovery, and this type of seizures should be treated.

Infantile spasms are better viewed as a type of seizure rather than an epilepsy syndrome, and are also discussed in questions 24 and 25. The triad of infantile spasms, hypsarrhythmia, and developmental arrest is known as West syndrome. This condition has been associated with multiple etiologies, such as hypoxic–ischemic injuries, brain malformations or structural abnormalities, congenital or acquired infections, chromosomal abnormalities, and inborn errors of metabolism. Infantile spasms are frequent in patients with tuberous sclerosis and this condition should be considered in this setting. Every patient presenting with infantile spasms should have an appropriate, thorough workup to look for the cause, including a brain MRI. In close to 30% of the cases, no specific etiology is found, and these cases are considered cryptogenic.

ACTH is commonly used for the treatment of infantile spasms. ACTH should be used carefully, given its potential side effects, which include hypertension, hyperglycemia, weight gain, electrolyte abnormalities, risk of infections, risk of avascular necrosis, and gastrointestinal bleeding. Vigabatrin may also be used for the treatment of infantile spasms, especially in patients with tuberous sclerosis. Vigabatrin should be used with caution as it carries the risk of retinal toxicity.

Bradley WG, Daroff RB, Fenichel GM, et al. Neurology in Clinical Practice, 5th ed. Philadelphia, PA: Elsevier; 2008.

Korff CM, Nordli DR. Epilepsy syndromes in infancy. Pediatr Neurol. 2006; 34:253–263.

46. d

This patient has Aicardi syndrome, which is a rare genetic disorder, usually associated with an X-linked dominant pattern of inheritance. Aicardi syndrome is characterized by the presence of infantile spasms, chorioretinal lacunae, and agenesis of the corpus callosum. Being an X-linked dominant disorder, it is encountered predominantly in girls, as the mutation is lethal in males. This syndrome is associated with various nonspecific ocular malformations, such as cataracts, microphthalmia, retinal detachment, and hypoplastic papilla. The presence of chorioretinal lacunae is pathognomonic for this syndrome. EEG shows multiple epileptiform abnormalities, such as burst suppression pattern with asynchrony between the two hemispheres and a disorganized background. West syndrome is described in question 45. Ohtahara syndrome is described in question 49. Dravet syndrome is described in question 48. Doose syndrome is described in question 47.

Rosser T. Aicardi Syndrome. Arch Neurol. 2003; 60:1471–1473.

47. e

This patient has Doose syndrome or myoclonic–astatic epilepsy. Typical onset is between 1 and 5 years of age. Children are normal prior to the onset of seizures, and many continue to have normal cognitive development. Seizures are predominantly generalized with myoclonic or astatic components, in which the patient loses postural tone and falls, sometimes resulting in injuries. There may be other seizure types, such as absence, generalized tonic-clonic (GTC), and tonic seizures, and/or nonconvulsive status epilepticus. The EEG demonstrates interictal bilateral synchronous irregular 2- to 3-Hz spike and wave complexes along with parietal rhythmic θ-activity. Myoclonic seizures are associated with irregular spikes and polyspikes. There may be a genetic predisposition, and a family history of epilepsy or abnormal EEGs is frequent.

Even though many patients remain normal, some have severe developmental delay and intractable seizures, and the prognosis may be variable. Valproic acid is commonly prescribed. Ethosuximide may help with absence seizures. Levetiracetam and ketogenic diet have also been reported to be beneficial in some cases.

Dravet syndrome is described in question 48. Ohtahara syndrome is described in question 49. Benign myoclonic epilepsy of infancy is described in question 50. Generalized epilepsy with febrile seizures plus (GEFS+) is described in question 7.

Bradley WG, Daroff RB, Fenichel GM, et al. Neurology in Clinical Practice, 5th ed. Philadelphia, PA: Elsevier, 2008.

Korff CM, Nordli DR. Epilepsy syndromes in infancy. Pediatr Neurol. 2006; 34:253–263.

48. a

This patient has Dravet syndrome or severe myoclonic epilepsy of infancy. This is a severe epilepsy syndrome, in which the patient has frequent seizures and various seizure types. The typical initial presentation is a febrile seizure (FS) in the first year of life; later, these patients develop other seizure types, including myoclonias, atypical absences, and tonic and tonic–clonic seizures, which could be generalized and/or unilateral. Given the initial presentation with an FS, the diagnosis may be delayed. Males are more affected than females, and there may be a family history of epilepsy or abnormal EEGs. In fact, Dravet syndrome may lie at the most severe end of the spectrum of generalized epilepsy with febrile seizures plus (GEFS+) and may commonly be associated with a mutation in the sodium channel SCN1A. The EEG may be normal initially in the interictal period, later showing generalized spike–wave complexes as well as focal and multifocal spikes. Developmental delay is the rule and neurologic abnormalities are common. The prognosis is poor, seizures are difficult to control, and there is sensitivity to hyperthermia. Treatment options include valproic acid, topiramate, zonisamide, and ketogenic diet. Importantly, treatment with phenobarbital, phenytoin, carbamazepine, and lamotrigine may exacerbate the seizures.

Ohtahara syndrome is described in question 49. Benign myoclonic epilepsy of infancy is described in question 50. Generalized epilepsy with febrile seizures plus (GEFS+) is described in question 7. Doose syndrome is described in question 47.

Bradley WG, Daroff RB, Fenichel GM, et al. Neurology in Clinical Practice, 5th ed. Philadelphia, PA: Elsevier; 2008.

Korff CM, Nordli DR. Epilepsy syndromes in infancy. Pediatr Neurol. 2006; 34:253–263.

49. b

This patient has Ohtahara syndrome, also known as early infantile epileptic encephalopathy (EIEE). This is a rare severe neurologic condition in which seizures begin during early infancy (between 1 day and 3 months of age). Patients have epileptic tonic spasms occurring multiple times per day. The EEG typically shows a burst suppression pattern that is present during wakefulness or sleep. This is a catastrophic epileptic encephalopathy with intractable seizures and a very poor prognosis. In one series, 25% of patients died before 2 years of age. All survivors have severe disabilities and developmental impairment.

Dravet syndrome is described in question 48. Benign myoclonic epilepsy of infancy is described in question 50. Generalized epilepsy with febrile seizures plus (GEFS+) is described in question 7. Doose syndrome is described in question 47.

Bradley WG, Daroff RB, Fenichel GM, et al. Neurology in Clinical Practice, 5th ed. Philadelphia, PA: Elsevier; 2008.

Korff CM, Nordli DR. Epilepsy syndromes in infancy. Pediatr Neurol. 2006; 34:253–263.

50. c

This patient has benign myoclonic epilepsy of infancy (BMEI). This condition affects males more than females, between the ages of 4 months and 3 years. It is characterized by the presence of brief myoclonic seizures, which are easily treatable. These myoclonias are brief (1–3 seconds) and usually isolated, and are more prominent during drowsiness, photostimulation, and external stimulation. Unlike infantile spasms, the myoclonic seizures of BMEI do not occur in long series/clusters. During a myoclonic seizure, the EEG shows generalized spikes and waves or polyspikes and waves. The interictal EEG is normal. Neuroimaging is usually normal. Seizures respond well to valproic acid, and the prognosis is generally good with spontaneous resolution of seizures in less than a year. Neuropsychologic outcome is favorable, although a small minority of patients may have mild mental retardation.

Dravet syndrome is described in question 48. Ohtahara syndrome is described in question 49. Generalized epilepsy with febrile seizures plus (GEFS+) is described in question 7. Doose syndrome is described in question 47.

Bradley WG, Daroff RB, Fenichel GM, et al. Neurology in Clinical Practice, 5th ed. Philadelphia, PA: Elsevier; 2008.

Korff CM, Nordli DR. Epilepsy syndromes in infancy. Pediatr Neurol. 2006; 34:253–263.

51. b

This patient has benign neonatal seizures. In this syndrome, full-term, otherwise healthy, newborns develop seizures around day 5 of life (also referred to as “fifth day fits”), which are partial clonic seizures that may be unilateral and/or symmetric and may migrate to other regions of the body. These seizures are frequently associated with apneic spells. The EEG is normal, but may demonstrate the “θ pointu alternant” pattern, characterized by discontinuous, asynchronous, unreactive θ-activity with intermixed sharp waves. Patients are neurologically normal. In general, there is no need for treatment with antiepileptic agents, and seizures resolve spontaneously by 4 to 6 weeks of age.

Benign neonatal seizures and benign familial neonatal seizures should be diagnoses of exclusion, and workup to rule out symptomatic seizures is indicated. Benign familial neonatal seizures is an autosomal dominant disorder, characterized by seizures in the first few days of life, which resolve spontaneously within few weeks. Genetic linkage studies have mapped two disease loci, both associated with mutations in voltage-gated potassium channels, in the genes KCNQ2 on chromosome 20 and KCNQ3 on chromosome 8.

West syndrome is described in question 45. Aicardi syndrome is discussed in question 46. Ohtahara syndrome is described in question 49. Benign myoclonic epilepsy of infancy is described in question 50.

Bradley WG, Daroff RB, Fenichel GM, et al. Neurology in Clinical Practice, 5th ed. Philadelphia, PA: Elsevier; 2008.

Browne TR, Holmes GL. Handbook of Epilepsy, 4th ed. Philadelphia, PA: Lippincott Williams & Wilkins; 2008.

52. c

This child presents with an idiopathic occipital epilepsy, more specifically early-onset childhood occipital epilepsy or Panayiotopoulos syndrome. In this condition, the seizures begin between 4 and 8 years of age (with a peak incidence at 4–5 years), and are characterized by tonic eye deviation and vomiting. Visual auras are reported during wakefulness, characterized by elementary or complex visual hallucinations and illusions. Partial or generalized tonic-clonic (GTC) seizures may occur during sleep; in fact, in the majority of children, seizures occur predominantly or exclusively in sleep. The EEG shows high-voltage occipital spikes in 1- to 3-Hz bursts, which disappear with eye opening and reappear with eye closure or darkness. Treatment is generally not required. The prognosis is good, and this condition resolves within several years.

Late-onset childhood occipital epilepsy or Gastaut type occurs in older children at a mean age of 8 years (between the ages of 4 and 13 years) and consists of brief seizures with visual manifestations followed by hemiclonic convulsions and in some cases a postictal migraine. The EEG is similar to that seen in Panayiotopoulos syndrome. The prognosis is variable in the Gastaut type, but most patients have a benign course. However, pharmacologic therapy may be needed and seizures may be difficult to control in some cases.

Ohtahara syndrome is discussed in question 49. Dravet syndrome is discussed in question 48. Doose syndrome is discussed in question 47.

Bradley WG, Daroff RB, Fenichel GM, et al. Neurology in Clinical Practice, 5th ed. Philadelphia, PA: Elsevier; 2008.

Browne TR, Holmes GL. Handbook of Epilepsy, 4th ed. Philadelphia, PA: Lippincott Williams & Wilkins; 2008.

53. d

This patient has Lennox–Gastaut syndrome. This syndrome is characterized by the triad of seizures of multiple types, EEG with diffuse slow (1.5–2 Hz) spike–wave complexes, and mental retardation. The onset is between the ages of 1 to 8 years, with most children presenting at the age of 3 to 5 years. Less than half of these patients will have normal cognitive function before the onset of seizures, eventually deteriorating after the onset of seizures leading to severe psychomotor retardation. About 60% of the cases have an identified cause, but some are cryptogenic. Patients with Lennox–Gastaut syndrome will develop various seizure types, including atypical absence, tonic, atonic, myoclonic, and tonic–clonic seizures. Valproic acid and clonazepam are frequently used. Other medications that could be given include lamotrigine, felbamate, topiramate, and vigabatrin. Ketogenic diet may be considered. These seizures are often refractory to therapy.

Panayiotopoulos syndrome is described in question 52. West syndrome is described in question 45. Landau–Kleffner syndrome is described in question 54. Question 43 contains a description of seizures arising from the mesial temporal lobe.

Bradley WG, Daroff RB, Fenichel GM, et al. Neurology in Clinical Practice, 5th ed. Philadelphia, PA: Elsevier; 2008.

Fenichel GM. Clinical Pediatr Neurol: A Signs and Symptoms Approach, 6th ed. Philadelphia, PA: Saunders Elsevier; 2009.

54. c

This patient has Landau–Kleffner syndrome, also known as acquired epileptic aphasia. This syndrome is characterized by an acquired aphasia associated with epileptiform abnormalities on EEG and seizures of various types. The age of onset is between 2 and 11 years, with a peak onset between 5 and 7 years; these children may initially present with word deafness in the setting of normal hearing. The disorder of language progresses and both a receptive and expressive aphasia may eventually occur. The seizures are of various types, including atypical absence, myoclonic, tonic, and tonic–clonic. Furthermore, a small minority of patients do not have a history of clinical seizures. The EEG demonstrates multifocal cortical spikes, predominantly in the temporal and parietal lobes, most frequently bilaterally. Antiepileptic agents such as valproic acid and lamotrigine are usually effective in controlling the seizures. Recovery of speech on the other hand is variable, with some patients having significant improvement, but others not. Corticosteroids have been tried with variable success.

Panayiotopoulos syndrome is described in question 52. West syndrome is described in question 45. Lennox–Gastaut syndrome is described in question 53. Question 43 contains a description of seizures arising from the mesial temporal lobe.

Bradley WG, Daroff RB, Fenichel GM, et al. Neurology in Clinical Practice, 5th ed. Philadelphia, PA: Elsevier; 2008.

Fenichel GM. Clinical Pediatr Neurol: A Signs and Symptoms Approach, 6th ed. Philadelphia, PA: Saunders Elsevier; 2009.

55. d

This patient has autosomal dominant nocturnal frontal lobe epilepsy (ADNFLE). These seizures begin in childhood and frequently persist into adult life. Patients with ADNFLE present with bizarre episodic behaviors in the context of hypermotor seizures, that is, hyperkinetic seizures with prominent motor phenomena, such as thrashing and jerking. These seizures occur during non-REM sleep, and patients may experience sudden awakenings with motor manifestations. Some patients will be conscious and report auras with epigastric, sensory, or psychic components. Because of the unusual appearance of these seizures, they are often mistaken for psychogenic nonepileptic seizures (“pseudoseizures”) or sleep-related disorders. The interictal EEG is usually normal, and the diagnosis is based on capturing the seizures on video EEG. These seizures usually respond well to carbamazepine or oxcarbazepine. Mutations in the genes that encode subunits of the nicotinic acetylcholine receptors, CNRNA4 and CHRNB2, have been detected.

Electrical status epilepticus during slow-wave sleep (ESES) presents in children between ages 1 and 12 years (peak around 4–5 years) with psychomotor retardation and multiple seizure types that occur more often during sleep. The diagnosis is made with the EEG showing slow spike–wave complexes occurring during non-REM sleep occupying at least 85% of the slow-wave sleep time. This disorder has been linked to Landau–Kleffner syndrome (see question 54). Although there is an overlap between these two syndromes, children with ESES present with a more global regression and seizures that may be more difficult to treat.

Lennox–Gastaut syndrome is described in question 53. Landau–Kleffner syndrome is described in question 54. Panayiotopoulos syndrome is described in question 52.

Bradley WG, Daroff RB, Fenichel GM, et al. Neurology in Clinical Practice, 5th ed. Philadelphia, PA: Elsevier; 2008.

Browne TR, Holmes GL. Handbook of Epilepsy, 4th ed. Philadelphia, PA: Lippincott Williams & Wilkins; 2008.

56. b, 57. d, 58. a, 59. a

Lateralizing semiological signs and symptoms during epileptic seizures are helpful to predict the side of the seizure focus, and this may have implications on the correct classification of the patient’s epilepsy and for presurgical evaluation in patients whose seizures are difficult to control with AEDs. Auras preceding a seizure represent focal discharges, which activate the area responsible for generation of the patient’s aura at the beginning of the clinical (but not necessarily the electrographic) seizure. Thus, auras may provide a clue with regard to the seizure origin. Some seizures do not start with an aura, but other semiological signs may point toward the origin of these seizures.

Eyes and head version, characterized strictly by a forced and involuntary movement leading to an unnatural position of the head toward one side, are associated with a seizure focus in the contralateral hemisphere, and more specifically in the frontal region in the frontal eye fields and motor areas anterior to the precentral gyrus. The association of version with the contralateral hemisphere is more robust if the version occurs immediately prior to the secondarily generalized tonic–clonic phase. Therefore the patient in question 56 has a seizure focus in the right frontal region.

Question 57 depicts a case of asymmetric tonic posturing during a seizure, which is characteristic of seizures arising from the supplementary motor area (SMA). This specific case describes the “figure of 4 sign,” in which the extended arm is contralateral to the seizure focus. Therefore, this patient’s seizures originate in the left SMA (see question 44).

Unilateral dystonic hand/arm posture during a seizure has important lateralizing value in temporal lobe epilepsies, suggesting that the seizure arises from the temporal lobe contralateral to the dystonic upper extremity. The suggested hypothesis is that the discharges originate in the hippocampus and amygdala, spreading via the fornix and through the basal ganglia, more specifically the ventral striatum, pallidum, and anterior cingulate gyrus. Patients with dystonic posture of one upper limb and automatisms in the opposite upper limb have seizures originating in the temporal lobe ipsilateral to the automatisms. In question 58, the seizure focus is in the right temporal lobe.

Gelastic seizures are characterized by uncontrollable episodes of laughter that occur in clusters, as depicted in question 59. These seizures are rare, and commonly originate in the hypothalamus, more specifically associated with hypothalamic hamartomas, although gelastic seizures arising from various cortical areas mainly in the frontal and temporal lobe have been well described in the literature. Hypothalamic hamartoma is a congenital malformation that may be asymptomatic in some cases, but commonly presents with precocious puberty and seizures. Seizures may progress and the patient may manifest other seizure types.

Harvey AS, Freeman JL. Epilepsy in hypothalamic hamartoma: clinical and EEG features. Semin Pediatr Neurol. 2007; 14(2):60–64.

Loddenkemper T, Kotagal P. Lateralizing signs during seizures in focal epilepsy. Epilepsy & Behavior. 2005; 7:1–17.

60. c

This patient has phenytoin toxicity, and free and total phenytoin levels should be checked. CT and MRI of the brain may need to be performed to rule out a new cerebrovascular event; however, the symptoms most likely correlate with the introduction of the two antimicrobial medications resulting in phenytoin toxicity. Given the patient’s symptoms, the most likely cause can be determined by assessing the serum levels of phenytoin. There is no evidence to suggest a new seizure and therefore an EEG is not indicated. Urinalysis and urine culture may need to be obtained to rule out an infectious process; however, it is unlikely to be the cause of this patient’s symptoms.

Phenytoin is a widely used medication for seizures. However, due to its narrow therapeutic index, nonlinear kinetics, and multiple interactions with other medications, toxicity can occur in patients treated with this medication. Phenytoin can be given orally or intravenously. It binds extensively to albumin (almost 90%); only unbound phenytoin is pharmacologically active. Medications (such as sulfamethoxazole) that displace phenytoin from albumin may increase free levels, producing manifestations of toxicity.

Phenytoin is metabolized by hepatic cytochrome (CYP) P450 enzymes to inactive metabolites, which are then excreted in the urine. Phenytoin follows zero-order kinetic metabolism (discussed in questions 26–28).

There is a long list of medications that could potentially interact with the metabolism of phenytoin, either increasing or decreasing its serum levels. Fluconazole and trimethoprim are substrates of the same CYP 450 pathway, and can therefore raise levels of this antiepileptic drug.

The clinical manifestations of phenytoin toxicity are usually neurologic in nature, and depending on the drug level, range from dizziness, nystagmus, ataxia, and other manifestations of cerebellar dysfunction to lethargy, confusion, and coma.

Bradley WG, Daroff RB, Fenichel GM, et al. Neurology in Clinical Practice, 5th ed. Philadelphia, PA: Elsevier; 2008.

Craig S. Phenytoin poisoning. Neurocrit Care. 2005; 3:161–170.

French JA, Pedley TA. Initial management of epilepsy. N Engl J Med. 2008; 359:166–176.

Toledano R, Gil-Nagel A. Adverse effects of antiepileptic drugs. Semin Neurol. 2008; 28:317–327.

61. e, 62. a

This patient had a simple febrile seizure (FS). FS is defined as a seizure that occurs in association with a febrile illness in the absence of CNS infection or acute electrolyte imbalance in children without prior afebrile seizures. These occur commonly between 6 months and 5 years of age, with a peak incidence at 18 months. FS can be simple or complex. Most FS represent simple FS, that is, generalized, usually isolated, seizures of relatively brief duration lasting less than 15 minutes. Complex FS on the other hand are prolonged, lasting more than 15 minutes, have focal features, and may occur multiple times in the course of the same febrile illness, or within the same 24-hour period.

A prior history of a complex FS is a known risk factor for subsequent development of epilepsy, whereas a family history of FS does not increase the risk of subsequent epilepsy. Interestingly, a prior history of a complex FS has not been found to be associated with a higher risk for recurrence of FS.

Risk factors to develop a first FS include a first- or second-degree relative with a history of FS, developmental delay, neonatal nursery stay for more than 30 days, and attendance of day care. Most patients will have only one FS without recurrence and without subsequent development of epilepsy. Recurrent FS may occur, especially in the presence of recognizable risk factors, which include a family history of FS, age younger than 18 months at the time of the first FS, lower peak temperature, and shorter duration of fever prior to the FS. Simple or complex FS carry similar risk of recurrence.

The risk of developing epilepsy following a single simple FS is not substantially different than the risk in the general population. On the other hand, patients with complex FS may be at risk of developing epilepsy in the future. The risk is even higher when the complex FS was very prolonged in duration (i.e., febrile status epilepticus). Other risk factors to develop epilepsy include the presence of a neurodevelopmental abnormality and/or family history of epilepsy. Importantly, a family history of FS does not predispose the patient to subsequent development of epilepsy.

The evaluation of patients with FS should be targeted at assessing the cause of the seizure and ruling out CNS infections or abnormalities. The exact pathophysiology of FS remains unclear. A relationship between prolonged FS and mesial temporal lobe sclerosis has been proposed; however, this association is controversial and has not conclusively been confirmed in prospective and population-based studies, which are ongoing.

Long-term prophylaxis and AEDs are not indicated for FS, but can be considered in some cases, especially if a high risk of recurrence exists. In these cases, phenobarbital or valproic acid can be considered on a case-by-case basis. Short-term prophylaxis may also be needed in some cases, and should be focused on temperature control during febrile illness and use of benzodiazepines, such as rectal diazepam gel.

Shinnar S, Glauser TA. Febrile Seizures. J Child Neurol. 2002; 17:S44–S52.

63. b

This patient has Unverricht Lundborg syndrome, one of the progressive myoclonic epilepsies. This group of disorders is characterized by myoclonic epilepsy and progressive neurologic deterioration, and includes Unverricht Lundborg syndrome, Lafora body disease, myoclonic epilepsy with ragged red fibers (MERFF), sialidosis, and neuronal ceroid lipofuscinosis. Unverricht Lundborg syndrome, also known as Baltic myoclonic epilepsy, is an autosomal recessive condition associated with mutations in the gene EPM1 located on the chromosomal locus 21q22.3, which encodes for cystatin B, a cysteine protease inhibitor associated with the initiation of apoptosis. Patients with Unverricht Lundborg syndrome present between 6 and 15 years of age with stimulus-sensitive myoclonus, which is action related and worsens over time. Eventually, they develop various seizure types, including absences, focal motor, or generalized tonic–clonic seizures. These patients will deteriorate neurologically, presenting with ataxia, tremor, and intellectual decline. MRI is usually normal, and EEG shows generalized spike–waves and polyspikes. Treatment options include valproic acid, clonazepam, levetiracetam, and zonisamide. Certain AEDs may worsen the seizures, including phenytoin, carbamazepine, oxcarbazepine, vigabatrin, tiagabine, gabapentin, and pregabalin. These patients may worsen progressively, but in some cases, the disease stabilizes over the years.

Lafora body disease is discussed in question 66. Sialidosis is discussed in question 65. Juvenile myoclonic epilepsy is discussed in questions 19 and 20, and neuronal ceroid lipofuscinosis is discussed in Chapter 14.

Bradley WG, Daroff RB, Fenichel GM, et al. Neurology in Clinical Practice, 5th ed. Philadelphia, PA: Elsevier; 2008.

Delgado-Escueta AV, Ganesh S, Yamakawa K. Advances in the genetics of progressive myoclonus epilepsy. Am J Med Genet. 2001; 106:129–138.

64. c

This patient has myoclonic epilepsy with ragged red fibers (MERFF), which is a mitochondrial disorder.

The patient has various characteristics suggestive of a mitochondrial disorder, including migraines, short stature, ataxia, cognitive impairment, deafness, epilepsy, and elevated lactate. He has generalized proximal weakness suggesting a myopathy, and the muscle biopsy in Figure 5.6 demonstrates ragged red fibers, which supports the diagnosis.

Mitochondrial disorders are a heterogeneous group of disorders that can affect both the peripheral and central nervous system. MERRF is a mitochondrial disorder that usually starts in the second or third decade of life, and is maternally inherited (like other disorders of mitochondrial DNA). CSF studies will show elevation of pyruvate and lactate, and the serum creatine kinase may be elevated. MRI of the brain usually demonstrates cerebral atrophy. Point mutations in mitochondrial DNA have been detected in this disorder.

Mutation affecting cystatin B is seen with EPM1 mutations in Unverricht Lundborg syndrome. Cherry red spot is seen in various conditions, including sialidosis, which is another progressive myoclonic epilepsy. EPM2Amutation is seen in Lafora body disease, in which Lafora bodies are also detected on skin biopsy.

Delgado-Escueta AV, Ganesh S, Yamakawa K. Advances in the genetics of progressive myoclonus epilepsy. Am J Med Genet. 2001; 106:129–138.

Schmiedel J, Jackson S, Schafer J, et al. Mitochondrial cytopathies. J Neurol. 2003; 250:267–277.

65. c

This patient has a progressive myoclonic epilepsy (PME), more specifically sialidosis. There are two types of sialidosis that can cause PME: Type I is caused by a deficiency of α-neuraminidase and presents in adolescents and adults with action myoclonus, and slowly progressive ataxia, tonic–clonic seizures, and vision loss. These patients do not have mental deterioration or dysmorphism, and characteristically, the fundoscopic examination demonstrates a cherry red spot. Type II is caused by deficiency of N-acetyl neuraminidase and β-galactosialidase, and begins between the neonatal period and the second decade of life. These patients have myoclonus, along with coarse facial features, corneal clouding, hepatomegaly, skeletal dysplasia, and learning disabilities. The sialidoses are autosomal recessive, and the gene implicated is NEU1 in chromosome 6p21.3. The diagnosis is confirmed with the detection of high urinary sialyloligosaccharides and by confirmation of the lysosomal enzyme deficiency in leukocytes or cultured fibroblasts.

Unverricht Lundborg syndrome is discussed in question 63. Lafora body disease is discussed in question 66. Juvenile myoclonic epilepsy is discussed in questions 19–20, and neuronal ceroid lipofuscinosis is discussed in Chapter 14.

Delgado-Escueta AV, Ganesh S, Yamakawa K. Advances in the genetics of progressive myoclonus epilepsy. Am J Med Genet. 2001; 106:129–138.

Shahwan A, Farrel M, Delanty N. Progressive myoclonic epilepsies: a review of genetic and therapeutic aspects. Lancet Neurol. 2005; 4:239–248.

66. e

This patient has Lafora body disease, which is an autosomal recessive disorder associated with a mutation in the gene EPM2A on chromosome 6q, encoding laforin, a ribosomal protein with undetermined function. Patients with Lafora body disease present between 12 and 17 years of age. These patients have seizures of various types, including myoclonus, atypical absences, atonic, complex partial, and occipital seizures with transient blindness and visual hallucinations. These patients also have dysarthria, ataxia, as well as emotional disturbance, and cognitive decline leading to dementia. EEG shows an evolution, with multiple spike–wave discharges at the beginning, but progressively over months or years, the background deteriorates and multifocal epileptiform abnormalities appear, mainly in the occipital regions, in addition to generalized bursts. Lafora bodies are PAS-positive intracellular polyglucosan inclusion bodies found in neurons, cardiac muscle, skeletal muscle, hepatocytes, and sweat gland duct cells, making it possible to detect these bodies in skin biopsy specimens. Most patients die within 10 years of onset, and the treatment remains palliative.

Mutation in cystatin B and EPM1 mutation are seen in Unverricht Lundborg syndrome. Cherry red spot in a patient with progressive myoclonic epilepsy (PME) suggests sialidosis. Ragged red fibers on muscle biopsy are seen in myoclonic epilepsy with ragged red fibers.

Delgado-Escueta AV, Ganesh S, Yamakawa K. Advances in the genetics of progressive myoclonus epilepsy. Am J Med Genet. 2001; 106:129–138.

Shahwan A, Farrel M, Delanty N. Progressive myoclonic epilepsies: a review of genetic and therapeutic aspects. Lancet Neurol. 2005; 4:239–248.

67. b

In Rasumussen’s encephalitis, seizures are not typically well controlled with AED monotherapy.

Rasmussen’s encephalitis is an inflammatory condition that affects one hemisphere, and is characterized by focal seizures, often epilepsia partialis continua, hemiparesis, and progressive neurologic deterioration. This condition most frequently affects children, though adolescents and adults may also be affected. The pathogenesis is not well understood; however, it is known that it is an inflammatory condition, and an antibody has been detected targeted against the GluR3 subunit of the AMPA receptor, which is a glutamate receptor. Histopathologic findings demonstrate perivascular cuffs of lymphocytes and monocytes, as well as glial nodules in the gray and white matter, and the continuous neuron loss leaves areas of spongy tissue degeneration. MRI demonstrates cortical atrophy that is progressive and focal areas of white matter hyperintensity. Seizures are usually intractable with AEDs. Anti-inflammatory agents such as corticosteroids and other immune modulating treatments including IV immune globulin and plasmapheresis have been tried with some promise, but variable success. Hemispherectomy is often needed, providing the possibility of cure of the seizures. Rasmussen’s syndrome is also discussed in question 8.

Bradley WG, Daroff RB, Fenichel GM, et al. Neurology in Clinical Practice, 5th ed. Philadelphia, PA: Elsevier; 2008.

Granata T. Rasmussen’s syndrome. Neurol Sci. 2003; 24:S239–243.

68. e

Absorption, distribution, metabolism, and excretion of AEDs are altered in the elderly. These patients tend to have lower gastric acidity, making the weakly basic drugs less easily absorbed and weakly acidic drugs more easily absorbed. Gastric emptying may be slowed and intestinal villi height may be reduced, making the absorption surface smaller. Given that lean body mass decreases with aging, total body water mass decreases, making the volume of distribution of hydrophilic drugs smaller. The proportion of fat also decreases with age, reducing lipophilic drug distribution volume. The metabolism of drugs is also affected, and hepatic metabolism decreases, given the reduction in liver volume, hepatic flow, and bile flow. Renal blood flow and glomerular filtration rate are lower in the elderly, and therefore, renal excretion is also decreased. Hepatic synthesis of protein is lower, and the free fraction of drug that binds to protein tends to increase. Therefore the measurement of free drug levels is recommended if available for that specific drug. The therapeutic window is also smaller in elderly patients. These patients tend to have multiple comorbidities and are usually on multiple medications that could potentially interact with AEDs. Therefore, careful selection of drugs and monitoring for side effects are strongly recommended in this group of patients.

Jetter GM, Cavazos JE. Epilepsy in the Elderly. Semin Neurol. 2008; 28(3):336–341.

69. d

Valproic acid is a broad-spectrum antiepileptic agent that can be used in various seizure types. Its acts by blocking sodium channels, but it may have other mechanisms of action, including effects on GABAA receptors and T-type calcium channels. This medication is widely used; however, it has various side effects. These include weight gain, alopecia, tremor, gastrointestinal symptoms, such as nausea and vomiting, and pancreatitis. It can be hepatotoxic and even produce fulminant hepatic failure and hyperammonemia, leading to encephalopathy. It can be undesirable in women, as it may produce a polycystic ovarian syndrome with menstrual irregularities, and it is teratogenic, specifically producing neural tube defects. Valproic acid is an enzyme inhibitor, and may lead to interactions with other medications, elevating the levels of many medications, including other AEDs. Osteoporosis from vitamin D deficiency is postulated to occur from enzyme-inducing antiepileptic agents; however, vitamin D deficiency is also seen in patients taking valproic acid monotherapy.

Hyponatremia is not a side effect of valproic acid and is more commonly seen with carbamazepine and oxcarbazepine.

Bradley WG, Daroff RB, Fenichel GM, et al. Neurology in Clinical Practice, 5th ed. Philadelphia, PA: Elsevier; 2008.

French JA, Pedley TA. Initial management of epilepsy. N Engl J Med. 2008; 359:166–176.

Toledano R, Gil-Nagel A. Adverse effects of antiepileptic drugs. Semin Neurol. 2008; 28:317–327.

70. d

Many antiepileptic agents produce cognitive and behavioral side effects, with levetiracetam being notorious for doing so. This medication in general has few side effects and does not interact with other medications; however, it can produce significant behavioral problems and emotional lability, sometimes agitation and even aggressiveness. Levetiracetam should be used carefully in the elderly and in patients with psychiatric disorders, with alternative medications used when possible.

Of the antiepileptic agents mentioned in the options, the one that presents most behavioral side effects is levetiracetam. Lamotrigine is a broad-spectrum AED that has no significant adverse impact on cognition and behavior and has few side effects if titrated slowly, with skin rash being the main side effect. Valproic acid, besides being used for epilepsy, is also used as a mood stabilizer. Carbamazepine and phenytoin can produce fatigue, dizziness, nystagmus, ataxia, and sedation, but usually not the behavioral problems seen with levetiracetam.

Bradley WG, Daroff RB, Fenichel GM, et al. Neurology in Clinical Practice, 5th ed. Philadelphia, PA: Elsevier; 2008.

French JA, Pedley TA. Initial management of epilepsy. N Engl J Med. 2008; 359:166–176.

Toledano R, Gil-Nagel A. Adverse effects of antiepileptic drugs. Semin Neurol. 2008; 28:317–327.

71. e

This patient has acute angle-closure glaucoma, which is a potential adverse effect of topiramate. This medication acts on voltage-dependent sodium channels, and also has an effect on GABA receptors as well as glutamate receptors. It is a carbonic anhydrase inhibitor. Topiramate has been used as an AED as well as for the treatment of migraines. Potential side effects from topiramate include drowsiness, fatigue, ataxia, word-finding difficulties, difficulty concentrating, paresthesias, weight loss, metabolic acidosis, nephrolithiasis, and acute angle-closure glaucoma.

Valproic acid, levetiracetam, phenytoin, and lamotrigine have not been associated with acute angle-closure glaucoma.

French JA, Pedley TA. Initial management of epilepsy. N Engl J Med. 2008; 359:166–176.

Toledano R, Gil-Nagel A. Adverse effects of antiepileptic drugs. Semin Neurol. 2008; 28:317–327.

72. c

This patient had a first, likely unprovoked, seizure. The International League Against Epilepsy defines this as a seizure occurring in a person older than 1 month of age with no prior history of unprovoked seizures. This definition excludes neonatal seizures, febrile seizures (FS), or seizures in the setting of an acute precipitating cause. The risk of recurrence of seizures is influenced by various parameters, including history suggesting an underlying abnormal brain, a focal neurologic examination, an abnormal EEG, or evidence of an underlying cause on brain imaging studies. A practice parameter guideline has been published by the American Academy of Neurology on the basis of available evidence. At the time of that publication, there was evidence to support the use of a routine EEG and brain imaging with CT or MRI in patients presenting with a first unprovoked seizure. Blood tests such as blood count, glucose, and electrolytes may be helpful in certain cases and are almost always performed; however, there was not enough evidence to support such testing in these patients. The same holds for CSF analysis and toxicology screens, which may be helpful in certain situations.

Not every patient with a first unprovoked seizure needs to be treated. The decision to treat is a complex one, and depends on the risk of recurrence on the basis of the factors mentioned, the risk–benefit ratio of treatment, and other patient-specific considerations.

Haut SR, Shinnar S. Considerations in the treatment of a first unprovoked seizure. Semin Neurol. 2008; 28(3):289–296.

Krumholz A, Wiebe S, Gronseth G, et al. Practice parameter: evaluating an apparent unprovoked first seizure in adults (an evidence-based review). Neurology. 2007; 69:1996–2007.

73. d

This is an EEG showing posterior background with reactivity demonstrated after an eye closure. Normal background rhythm is seen in the posterior head regions, with α-frequency (between 8 and 13 Hz), and is usually present in normal people when they are awake, more prominent with eye closure, and attenuating with eye opening. Failure of attenuation with eye opening and reactivity with eye closure may be a sign of abnormality. Usually an α-frequency of 8 Hz is seen by 3 years of age in normal children, and this frequency increases with age. A voltage asymmetry of >50% between sides is abnormal. The posterior background is the first feature usually analyzed when reading an EEG.

There are no spikes or other epileptiform discharges seen in Figure 5.7 and therefore no evidence of an occipital seizure. The posterior background has α-frequencies (in this case around 9 Hz) and not δ-frequencies. This patient has good reactivity of the background rhythm, suggesting an awake patient. There are no sleep structures visualized.

Tatum WO, Husain AM, Benbadis SR, et al. Handbook of EEG Interpretation. Demos Medical Publishing, LLC; 2008.

74. c

α-frequency is between 8 and 13 Hz, and is usually present in normal people when they are awake, more prominent with eye closure, and attenuating with eye opening. Failure of attenuation with eye opening and reactivity with eye closure may be a sign of abnormality. Usually an α-frequency of 8 Hz is seen by 3 years of age in normal children, and this frequency increases with age.

β-frequencies are those greater than 13 Hz, are normal, but may be enhanced by benzodiazepines or barbiturates, and may increase with drowsiness and light sleep.

θ-frequencies are those between 4 and 7 Hz, and are present in the frontal and frontocentral regions of one-third of young adults. These frequencies are enhanced with focused concentration, during mental tasks, by hyperventilation, drowsiness, and sleep.

δ-frequencies are those lesser than 4 Hz, and can be seen in normal infants, in sleep, and sometimes in the elderly. Generalized δ frequencies may indicate nonspecific encephalopathy, and if focal, may indicate a structural lesion.

Bradley WG, Daroff RB, Fenichel GM, et al. Neurology in Clinical Practice, 5th ed. Philadelphia, PA: Elsevier; 2008.

Tatum WO, Husain AM, Benbadis SR, et al. Handbook of EEG Interpretation. Demos Medical Publishing, LLC; 2008.

75. c

EEG represents the recording along the scalp of electrical activity that is produced by the firing of pyramidal neurons within the cerebral cortex. EEG recording is based on differential amplification, in which the difference in voltage between two sites is compared with each recording channel. This recording helps in the evaluation and diagnosis of abnormalities in brain electrical activity, but is also useful in the evaluation of other neurologic conditions. Patients undergoing an evaluation for “spells” will often have an EEG to determine the presence or absence of epileptogenic activity. During a routine recording, various procedures are utilized to enhance potentially abnormal electrical activity. These so-called activation procedures include hyperventilation, photic stimulation, sleep deprivation prior to the EEG, and recording during sleep. Noxious stimulation is not part of the EEG activation procedures.

Hyperventilation may produce generalized slowing or no effect in normal subjects; however, in certain epilepsies, such as absence epilepsy, hyperventilation may activate epileptiform discharges and even seizures. Photic stimulation is performed with various frequencies, and may be helpful to induce epileptiform activity and seizures, most commonly myoclonic seizures, in individuals with photosensitive epilepsies. Sleep deprivation may also enhance epileptogenic activity. Furthermore, seizures may occur predominantly (or exclusively) during sleep in certain epilepsy syndromes, and EEG abnormalities may not be seen in the awake patient. Therefore an EEG may be incomplete without a sleep recording.

Tatum WO, Husain AM, Benbadis SR, et al. Handbook of EEG Interpretation, Demos Medical Publishing, LLC; 2008.

76. a

The EEG in Figure 5.8 shows diffuse slowing and triphasic waves, suggesting a metabolic encephalopathy. Triphasic waves represent a special type of continuous generalized slow activity, with characteristic slow waves that consist of three phases, beginning with a small negative (upward) wave, followed by a prominent positive (downward) wave, and ending in a negative (upward) wave. These waveforms typically occur in a bilaterally symmetric, bisynchronous fashion with an anterior-to-posterior lag. They are seen in various types of toxic and/or metabolic encephalopathies that involve altered states of consciousness, most commonly in hepatic encephalopathy. Other common causes include uremia, hypoglycemia, and electrolyte disturbances, such as hyponatremia or hypercalcemia.

Generalized periodic patterns are characterized by the appearance of generalized sharp wave discharges occurring in a periodic fashion. A generalized periodic pattern with a 1 Hz frequency is a characteristic finding of CJD. Although triphasic waves may sometimes be difficult to distinguish from generalized epileptiform activity, especially when occurring in prolonged runs, the EEG pattern seen in this figure would not be typical of a generalized seizure. Postcardiac arrest anoxia can produce various EEG patterns, including α-coma, burst suppression, and electrocerebral silence in the case of brain death. It should be noted that triphasic-appearing waves are also seen in anoxic brain injury. Infantile spasms are associated with the EEG finding of hypsarrhythmia (see questions 24 and 25).

Tatum WO, Husain AM, Benbadis SR, et al. Handbook of EEG Interpretation. Demos Medical Publishing, LLC; 2008.

77. a

This EEG shows burst suppression, characterized by bursts of electrical activity at regular intervals separated by intervals of no electrical activity represented by the flat EEG line. Burst suppression signifies severe bilateral cerebral dysfunction. The etiology of burst suppression is nonspecific. This pattern may be iatrogenic, such as in general anesthesia or in barbiturate coma for status epilepticus, among others. It may have a good prognosis in intoxications, but typically signifies a poor prognosis when seen in patients with a hypoxic–ischemic insult. Patients with burst suppression are comatose, and usually show no reactivity to activation procedures. Patients in status epilepticus intractable to antiepileptic agents may need to be treated with pharmacologically induced coma, in which case the therapy is initially targeted to burst suppression.

There are no triphasic waves, hypsarrhythmia, periodic lateralized epileptiform discharges, or generalized seizures in this EEG.

Tatum WO, Husain AM, Benbadis SR, et al. Handbook of EEG Interpretation. Demos Medical Publishing, LLC; 2008.

78. d

This patient has periodic lateralized epileptiform discharges (PLEDs) in the right hemisphere. PLEDs are sharply contoured waveforms with various morphologies that appear at regular periodic intervals every 1 or 2 seconds and are lateralized to one hemisphere only or to a single region in one hemisphere. These are seen in structural brain lesions, usually in the acute or subacute setting, such as a stroke, hemorrhage, infection (importantly HSV encephalitis), brain abscess, or tumor, and are usually transient, disappearing with time as the patient recovers from the acute event. Typically patients with PLEDs are encephalopathic with a diffusely slow EEG background. Usually PLEDs are not thought to represent ongoing ictal activity and are considered among the interictal phenomena/patterns. It is important to remember, however, that seizures may be seen in a significant number of patients who are found to have PLEDs on EEG.

Burst suppression is described in question 77. Triphasic waves are described in question 76. The figure does not show a generalized periodic pattern or sharp waves.

Tatum WO, Husain AM, Benbadis SR, et al. Handbook of EEG Interpretation. Demos Medical Publishing, LLC; 2008.

79. b, 80. e, 81. b

Sleep stages are separated into four stages: stage 1 (N1), stage 2 (N2), stage 3 (N3), and REM sleep. Stages 3 and 4 sleep (previously separated) are now combined together and called slow-wave sleep. Normal sleep consists of 4 to 6 cycles/night of non-REM (NREM) sleep, with each cycle followed by REM sleep. The first REM period is normally around 90 minutes after sleep onset. The electrographic and other characteristics of the sleep stages per current grading criteria are as follows:

N1 (previously stage 1) (5% of sleep time):

– Disappearance of occipital dominant α-rhythm

– Increasing θ-frequency in all regions

– Muscle artifact decreases

– Diphasic sharp waves maximal at vertex may occur (vertex waves)

– POSTS (positive occipital sharp transients of sleep) occur

N2 (previously stage 2) (45% of sleep time):

– Sleep spindles—bursts of 12- to 14-Hz activity maximal over central regions lasting less than 2 seconds; first appear around 2 months, but do not reach adult appearance until 2 years

– K complexes—high-voltage diphasic slow wave that may be preceded or followed by a sleep spindle, maximal over frontocentral regions

– Lower voltage, mixed-frequency background activity

– Vertex waves may still be seen

– POSTS may still be seen

– δ-activity less than 20% of sleep period

N3, or slow wave sleep (previously separated into stage 3 and stage 4) (20% to 25% of sleep time):

– High-amplitude δ-activity 20% to 50% of sleep period and has increased voltage

– Sleep spindles, K complexes, and POSTS rarely persist

– Stage 4 is no longer scored as an independent sleep stage

REM (20% to 25% of sleep time):

– Generally lower voltage, similar to stage 1

– In some individuals runs of α-activity may appear in occipital leads identical to α-rhythm in awake tracing

– Spontaneous REMs

– Tonic motor activity suppression on EMG leads

– Sawtooth waves in central regions

– The percentage of sleep spent in REM decreases with increasing age

– Selective serotonin reuptake inhibitors (SSRIs) reduce the length of time spent in REM

The suprachiasmatic nucleus in the anterior hypothalamus is known as the circadian pacemaker and regulates not only the sleep–wake cycle, but all biological circadian rhythms.

Iber C, Ancoli-Israel S, Chesson A, et al. The AASM Manual for the Scoring of Sleep and Associated Events: Rules, Terminology, and Technical Specification, 1st ed. Westchester, IL: American Academy of Sleep Medicine; 2007.

Levin K, Luders H. Comprehensive Clinical Neurophysiology. Philadelphia, PA: WB Saunders Company; 2000.

82. c

Obstructive sleep apnea (OSA) is diagnosed by the apnea-hypopnea index (AHI) observed during an overnight polysomnogram. The AHI counts how many apneas and hypopneas, on average, occur per hour. A similar although not exactly identical index of apneas and hypopneas is the so-called respiratory disturbance index (RDI), which in addition to apneas and hypopneas also accounts for respiratory-event-related arousals (RERAs). Apneas and hypopneas are characterized by complete cessation or reduction of airflow, respectively, for at least 10 seconds. Mild OSA is diagnosed by an AHI of 5 to 15/hour, moderate OSA by 15 to 30/hour, and severe OSA by more than 30/hour. In addition, apneas are associated with EEG arousal and oxygen desaturation of 2% to 4%.

Iber C, Ancoli-Israel S, Chesson A, et al. The AASM Manual for the Scoring of Sleep and Associated Events: Rules, Terminology, and Technical Specification, 1st ed. Westchester, IL: American Academy of Sleep Medicine; 2007.

83. c

This patient has obstructive sleep apnea (OSA). This syndrome is characterized by repetitive episodes of upper airway obstruction during sleep, associated with arousals or oxygen desaturations. Apneas should occur more than five times per hour and last at least 10 seconds, as detected by polysomnogram, which is a useful study for the diagnosis of this condition. During apneas, respiratory effort is present, differentiating this condition from central sleep apnea, in which there is no respiratory effort. OSAS is more frequently seen in obese patients and in those with small or crowded upper airways. The use of alcohol and sedatives may reduce airway tone, worsening OSAS.

Besides the clinical manifestations directly associated with OSAS, patients with this condition are at a higher risk of cardiovascular events. Patients with OSA are at higher risk to develop cardiac arrhythmias, hypertension, cor pulmonale, as well as myocardial infarctions and strokes. Management includes weight loss, avoidance of alcohol and sedatives, and use of positive airway pressure. Some patients may be candidates for upper airway surgery.

American Academy of Sleep Medicine. International Classification of Sleep Disorders, 2nd ed. Diagnostic and Coding Manual, Westchester, IL: American Academy of Sleep Medicine; 2005.

Bradley WG, Daroff RB, Fenichel GM, et al. Neurology in Clinical Practice, 5th ed. Philadelphia, PA: Elsevier; 2008.

Yaggi HK, Concato J, Kernan WN, et al. Obstructive sleep apnea as a risk factor for stroke and death. N Engl J Med. 2005; 353:2034–2041.

84. b

This patient has central sleep apnea syndrome (CSAS), characterized by recurrent episodes during which there is absent airflow along with a cessation of ventilatory effort during sleep. The clinical manifestations of CSAS are similar to those of obstructive sleep apnea (OSA), because patients will often experience insomnia, inability to maintain sleep, and excessive daytime sleepiness, as well as arousals, bradycardia or tachycardia, and desaturations during the apneas. The difference is that the main problem is a transient central cessation of respiratory drive, and airway obstruction does not occur. Polysomnogram is helpful in making the diagnosis, showing episodes of apnea, in which there is no respiratory effort. This type of sleep-disordered breathing is much less common than OSA. Given that there is no airway obstruction, surgical interventions are not indicated.

American Academy of Sleep Medicine. International Classification of Sleep Disorders, 2nd ed. Diagnostic and Coding Manual, Westchester, IL: American Academy of Sleep Medicine; 2005.

Bradley WG, Daroff RB, Fenichel GM, et al. Neurology in Clinical Practice, 5th ed. Philadelphia, PA: Elsevier; 2008.

85. d

This patient has a non-REM parasomnia, more specifically sleep terrors. Parasomnias have been classified into REM parasomnias, arousal disorders, sleep–wake transition disorders, and other parasomnias. Arousal disorders include confusional arousals, sleepwalking, and sleep terrors. The latter three are non-REM parasomnias arising from slow-wave sleep or stage III sleep (previously known as stage III and stage IV sleep; these two stages have more recently been combined into stage III sleep).

Sleep terrors are more common in children, usually between the ages of 5 and 7 years. These events are characterized by a sudden arousal with screaming or crying, associated with autonomic and behavioral manifestations of intense fear and some degree of confusion on awakening. Sleep terrors occur in the first third of the night. In nightmares, as opposed to sleep terrors, the patients usually remember the dream in detail, they occur in the last third of the night, and there is less autonomic activity. When awakened from nightmares, patients tend to have good intellectual function and are not confused.

American Academy of Sleep Medicine. International Classification of Sleep Disorders, 2nd ed. Diagnostic and Coding Manual, Westchester, IL: American Academy of Sleep Medicine; 2005.

Bradley WG, Daroff RB, Fenichel GM, et al. Neurology in Clinical Practice, 5th ed. Philadelphia, PA: Elsevier; 2008.

86. d

This patient has a non-REM parasomnia, specifically sleepwalking. Parasomnias have been classified into REM parasomnias, arousal disorders, sleep–wake transition disorders, and other parasomnias. Arousal disorders include confusional arousals, sleepwalking, and sleep terrors. The latter three are non-REM parasomnias arising from slow-wave sleep or stage III sleep (previously known as stage III and stage IV sleep; these two stages have more recently been combined into stage III sleep).

Sleepwalking consists of a series of complex behaviors resulting in walking during sleep. This condition occurs more commonly in children, but can present in adolescents and adults, and a positive family history has been reported in many cases. Sleepwalking occurs more commonly in the first third of the sleep, and patients have complex motor behaviors, with amnesia of the episode. These patients are difficult to arouse, and may become confused and exhibit violent behavior when this is attempted.

Confusional arousals are a non-REM parasomnia occurring in children, in which the patient is confused following an arousal from slow-wave sleep. The clinical presentation of this patient is not consistent with narcolepsy, nor with sleep terrors.

American Academy of Sleep Medicine. International Classification of Sleep Disorders, 2nd ed. Diagnostic and Coding Manual, Westchester, IL: American Academy of Sleep Medicine; 2005.

Bradley WG, Daroff RB, Fenichel GM, et al. Neurology in Clinical Practice, 5th ed. Philadelphia, PA: Elsevier; 2008.

87. a

This patient has had a nightmare, which is a complicated dream that becomes frightening toward the end. Nightmares should be differentiated from sleep terrors. Patients with nightmares usually recall the dream in detail, the event occurs in the last third of the night, and there is much less autonomic activity as compared to sleep terrors. When patients are awakened from nightmares, they tend to have good intellectual function and are not confused. Sleep terrors occur more commonly in children, and patients wake up screaming or crying, with prominent associated autonomic and behavioral manifestations of intense fear, as well as confusion on awakening. Sleep terrors occur in the first third of the night.

Confusional arousals are a non-REM parasomnia occurring in children, in which the patient is confused following an arousal from slow-wave sleep.

The clinical presentation of this patient is not consistent with REM sleep behavior disorder or a non-REM parasomnia.

American Academy of Sleep Medicine. International Classification of Sleep Disorders, 2nd ed. Diagnostic and Coding Manual, Westchester, IL: American Academy of Sleep Medicine; 2005.

Bradley WG, Daroff RB, Fenichel GM, et al. Neurology in Clinical Practice, 5th ed. Philadelphia, PA: Elsevier; 2008.

88. c

This patient has a REM parasomnia, specifically REM sleep behavior disorder (RBD).

Normally, there is atonia (loss of muscle tone) during REM sleep. RBD is characterized by intermittent loss of this normal REM atonia (REM without atonia) and by the appearance of complex motor activity during which the patient acts out dreams. Dream content is usually violent with associated movements (e.g., punching, kicking, and running). This can cause injuries to the patient or to the bed partner. RBD is seen in elderly patients, and may be associated with neurodegenerative conditions, more specifically α-synucleopathies such as Parkinson’s disease, multisystem atrophy, and DLB. Polysomnography is helpful in making the diagnosis, demonstrating the presence of REM without atonia.

American Academy of Sleep Medicine. International Classification of Sleep Disorders, 2nd ed. Diagnostic and Coding Manual, Westchester, IL: American Academy of Sleep Medicine; 2005.

Bradley WG, Daroff RB, Fenichel GM, et al. Neurology in Clinical Practice, 5th ed. Philadelphia, PA: Elsevier; 2008.

89. e

REM sleep behavior disorder (RBD) is characterized by REM without atonia and by the appearance of complex motor activity during which the patient acts out dreams. Dream content is usually violent with associated movements (e.g., punching, kicking, and running).

Sleep paralysis is a feature of narcolepsy and is characterized by a transient paralysis during sleep onset or on awakening; the patient is fully conscious during these events.

Hypnagogic hallucinations occur at sleep onset and are seen in patients with narcolepsy, and hypnopompic hallucinations occur on awakening from sleep.

A nightmare is a complicated dream that becomes frightening toward the end. Patients with nightmares usually recall the dream in detail, the event occurs in the last third of the night, and there is much less autonomic activity as compared to sleep terrors. When patients are awakened from nightmares, they tend to have good intellectual function and are not confused.

American Academy of Sleep Medicine. International Classification of Sleep Disorders, 2nd ed. Diagnostic and Coding Manual, Westchester, IL: American Academy of Sleep Medicine; 2005.

Bradley WG, Daroff RB, Fenichel GM, et al. Neurology in Clinical Practice, 5th ed. Philadelphia, PA: Elsevier; 2008.

90. c

This patient has Kleine–Levin syndrome, which is a type of recurrent hypersomnia. This condition is characterized by recurrent episodes of hypersomnia that typically occur weeks or months apart. The onset is in early adolescence, typically in males, and the episodes can last for several days and sometimes weeks, appearing many times per year. Patients sleep for prolonged periods of time, 18 to 20 hours, waking only to eat and void. During these episodes, patients may have disrupted behaviors, such as irritability, aggressiveness, confusion, hypersexuality, and a voracious appetite. Disturbance of social life is significant during these episodes. In between episodes, patient sleep well and behave normally.

This patient does not have idiopathic hypersomnia, narcolepsy, obstructive sleep apnea (OSA), or delayed sleep phase syndrome.

Idiopathic hypersomnia is characterized by long-term inability to obtain adequate sleep, in which the patients have excessive sleepiness and the etiology is thought to be abnormal neurologic control of the sleep–wake system. Patients complain of insomnia, they sleep for long periods of time, but the sleep is not refreshing. The diagnosis is made only when no other medical or psychiatric condition can explain the hypersomnia.

Narcolepsy is a disorder that is characterized by excessive sleepiness, cataplexy, sleep paralysis, and hypnagogic and hypnopompic hallucinations. Patients with narcolepsy suffer sleep attacks, in which the patient has an irresistible desire to fall asleep during inappropriate circumstances. These sleep attacks are short, lasting 15 to 30 minutes, and the patient feels refreshed afterward.

OSA is characterized by repetitive episodes of upper airway obstruction during sleep, associated with arousals or oxygen desaturations.

In delayed sleep phase syndrome, the major sleep episode is delayed in relation to the desired clock time, resulting in symptoms of sleep-onset insomnia and difficulty awakening. When these patients are able to sleep when they would like, there is no problem with falling asleep or waking up.

American Academy of Sleep Medicine. International Classification of Sleep Disorders, 2nd ed. Diagnostic and Coding Manual, Westchester, IL: American Academy of Sleep Medicine; 2005.

Bradley WG, Daroff RB, Fenichel GM, et al. Neurology in Clinical Practice, 5th ed. Philadelphia, PA: Elsevier; 2008.

91. d

This patient has narcolepsy with cataplexy. Narcolepsy is a disorder that is characterized by excessive sleepiness, cataplexy, sleep paralysis, and hypnagogic and hypnopompic hallucinations. Patients with narcolepsy suffer sleep attacks, in which the patient has an irresistible desire to fall asleep during inappropriate circumstances. These sleep attacks are short, lasting 15 to 30 minutes, and the patient feels refreshed afterward. Even though cataplexy is associated with narcolepsy, not all patients with narcolepsy have cataplexy. Cataplexy is characterized by episodes of sudden loss of tone of voluntary muscles, except respiratory and ocular muscles. During these attacks, the patients may fall and be unable to move, and deep tendon reflexes are decreased or absent. Patients have preserved consciousness. Cataplectic attacks are triggered by emotional events, such as laughter or anger.

This patient does not have pseudoseizures, nor gelastic seizures.

This patient does not have Kleine–Levin syndrome, which is a type of recurrent hypersomnia, characterized by recurrent episodes of hypersomnia that typically occur weeks or months apart. The episodes can last several days and sometimes weeks, appearing many times per year, and these patients sleep for prolonged periods of time, 18 to 20 hours, waking only to eat and void. During these episodes, patients may have disrupted behaviors, such as irritability, aggressiveness, confusion, hypersexuality, and a voracious appetite. In between episodes, patients sleep well and behave normally.

This patient does not have idiopathic hypersomnia, which is characterized by long-term inability to obtain adequate sleep, in which the patients have excessive sleepiness and the etiology is thought to be abnormal neurologic control of the sleep–wake system. Patients complain of insomnia, they sleep for long periods of time, but the sleep is not refreshing. The diagnosis is made only when no other medical or psychiatric condition can explain the hypersomnia.

American Academy of Sleep Medicine. International Classification of Sleep Disorders, 2nd ed. Diagnostic and Coding Manual, Westchester, IL: American Academy of Sleep Medicine;, 2005.

Bradley WG, Daroff RB, Fenichel GM, et al. Neurology in Clinical Practice, 5th ed. Philadelphia, PA: Elsevier; 2008.

92. a

Narcolepsy with cataplexy is thought to be associated with a loss of hypocretin neurons in the lateral hypothalamus, and hypocretin CSF levels are low in these patients.

During cataplectic attacks, patients have loss of muscle tone and hypo- or areflexia. Sleep paralysis is a feature that commonly accompanies this condition. The diagnosis of narcolepsy with cataplexy can be suspected on clinical grounds; however, polysomnogram and mean sleep latency test (MSLT) can provide information to support the diagnosis by detecting a mean sleep latency of less than 8 minutes and two or more sleep-onset REM periods (SOREMPs). γ-hydroxybutyrate has been approved for the treatment of sleepiness and cataplexy that is associated with narcolepsy. Other treatment options for cataplexy include tricyclic antidepressants or serotonin reuptake inhibitors.

American Academy of Sleep Medicine. International Classification of Sleep Disorders, 2nd ed. Diagnostic and Coding Manual, Westchester, IL: American Academy of Sleep Medicine; 2005.

Bradley WG, Daroff RB, Fenichel GM, et al. Neurology in Clinical Practice, 5th ed. Philadelphia, PA: Elsevier; 2008.

93. b

This patient has a circadian rhythm sleep disorder, specifically delayed sleep phase syndrome. Circadian rhythm disorders are characterized by a misalignment between the patient’s sleep pattern and the sleep pattern regarded as the societal norm. In delayed sleep phase syndrome, the major sleep episode is delayed in relation to the desired clock time, resulting in symptoms of sleep-onset insomnia and difficulty awakening. When these patients are able to sleep when they would like, there is no problem with falling asleep or waking up. This condition is more common in adolescents. Treatment may include chronotherapy or melatonin.

This patient does not have psychophysiologic insomnia, advanced sleep phase syndrome, jet lag syndrome, or shift work sleep disorder. In advanced sleep phase syndrome, the major sleep episode is advanced in relation to the desired clock time, resulting in excessive evening sleepiness and early sleep onset, as well as awakening earlier than desired. This condition is more common in the elderly. Jet lag syndrome is caused by the travel across multiple time zones. In shift work sleep disorder, patients experience symptoms of insomnia or excessive sleepiness that occur as transient phenomena in relation to work schedules. In psychophysiologic insomnia, patients worry about being unable to sleep and focus on their insomnia, which causes frustration and further inability to fall asleep.

American Academy of Sleep Medicine. International Classification of Sleep Disorders, 2nd ed. Diagnostic and Coding Manual, Westchester, IL: American Academy of Sleep Medicine; 2005.

Bradley WG, Daroff RB, Fenichel GM, et al. Neurology in Clinical Practice, 5th ed. Philadelphia, PA: Elsevier; 2008.

94. c

This patient has a circadian rhythm sleep disorder, specifically advanced sleep phase syndrome. Circadian rhythm sleep disorders are characterized by a misalignment between the patient’s sleep pattern and the sleep pattern regarded as the societal norm. In advanced sleep phase syndrome, the major sleep episode is advanced in relation to the desired clock time, resulting in excessive evening sleepiness and early sleep onset, as well as awakening earlier than desired. This condition is more common in the elderly.

This patient does not have psychophysiologic insomnia, delayed sleep phase syndrome, jet lag syndrome, or shift work sleep disorder. In psychophysiologic insomnia, patients worry about being unable to sleep and focus on their insomnia, which causes frustration and further inability to fall asleep. In delayed sleep phase syndrome, the major sleep episode is delayed in relation to the desired clock time, resulting in symptoms of sleep-onset insomnia and difficulty awakening. Jet lag syndrome is caused by the travel across multiple time zones. In shift work sleep disorder, patients experience symptoms of insomnia or excessive sleepiness that occur as a transient phenomena in relation to work schedules.

American Academy of Sleep Medicine. International Classification of Sleep Disorders, 2nd ed. Diagnostic and Coding Manual, Westchester, IL: American Academy of Sleep Medicine; 2005.

Bradley WG, Daroff RB, Fenichel GM, et al. Neurology in Clinical Practice, 5th ed. Philadelphia, PA: Elsevier; 2008.

95. e

This patient has a circadian rhythm sleep disorder, specifically shift work sleep disorder. In shift work sleep disorder, patients experience symptoms of insomnia or excessive sleepiness that occur as transient phenomena in relation to work schedules. Also, given the need for social interaction with people maintaining regular working hours, individuals with shift work sleep disorder may restrict their sleeping hours, worsening the symptoms of insomnia and sleepiness.

This patient does not have psychophysiologic insomnia, delayed sleep phase syndrome, advanced sleep phase syndrome, or jet lag syndrome. In psychophysiologic insomnia, patients worry about being unable to sleep and focus on their insomnia, which causes frustration and further inability to fall asleep. In delayed sleep phase syndrome, the major sleep episode is delayed in relation to the desired clock time, resulting in symptoms of sleep-onset insomnia and difficulty awakening. In advanced sleep phase syndrome, the major sleep episode is advanced in relation to the desired clock time, resulting in excessive evening sleepiness and early sleep onset, as well as awakening earlier than desired. This condition is more common in the elderly. Jet lag syndrome is caused by the travel across multiple time zones.

American Academy of Sleep Medicine. International Classification of Sleep Disorders, 2nd ed. Diagnostic and Coding Manual, Westchester, IL: American Academy of Sleep Medicine; 2005.

Bradley WG, Daroff RB, Fenichel GM, et al. Neurology in Clinical Practice, 5th ed. Philadelphia, PA: Elsevier; 2008.

96. a

This patient has psychophysiologic insomnia. Patients with this condition have difficulty falling asleep at the desired bedtime, with frequent awakening. These patients “try to fall asleep,” and are extremely concerned about and focus on their insomnia. There is frustration about being unable to initiate and maintain sleep. This worry about the need for sleep prevents these patients from falling asleep adequately, creating anxiety regarding sleepless. These patients do not meet criteria for a generalized anxiety disorder.

This patient does not have delayed sleep phase syndrome, advanced sleep phase syndrome, jet lag syndrome, or shift work sleep disorder. In delayed sleep phase syndrome, the major sleep episode is delayed in relation to the desired clock time, resulting in symptoms of sleep-onset insomnia and difficulty awakening. In advanced sleep phase syndrome, the major sleep episode is advanced in relation to the desired clock time, resulting in excessive evening sleepiness and early sleep onset, as well as awakening earlier than desired. This condition is more common in the elderly. Jet lag syndrome is caused by the travel across multiple time zones. In shift work sleep disorder, patients experience symptoms of insomnia or excessive sleepiness that occur as a transient phenomena in relation to work schedules.

American Academy of Sleep Medicine. International Classification of Sleep Disorders, 2nd ed. Diagnostic and Coding Manual, Westchester, IL: American Academy of Sleep Medicine; 2005.

Bradley WG, Daroff RB, Fenichel GM, et al. Neurology in Clinical Practice, 5th ed. Philadelphia, PA: Elsevier; 2008.

97. a

This patient has restless legs syndrome (RLS), which is characterized by an urge to move the legs that may be associated with abnormal sensations that may be difficult to describe. The urge to move the legs is worse at rest and in the evening or nighttime. The urge to move the legs is partially or completely relieved by movement. This condition occurs more commonly in women, and although the pathophysiology is not well known, it has been associated with abnormalities in dopaminergic pathways, basal ganglia abnormalities, and decreased ferritin levels. Patients with RLS can have sleep disturbance and insomnia.

Periodic limb movements (PLMs) are a polysomnographic finding, in which there are recurrent limb movements during non-REM sleep, more commonly of the lower extremities. PLMs can produce arousals and sleep fragmentation, leading to insomnia and excessive daytime sleepiness. PLMs are often seen in patients with RLS, but in this case, the most likely diagnosis on the basis of the clinical information is RLS.

This patient does not have psychophysiologic insomnia, delayed sleep phase syndrome, or advanced sleep phase syndrome. In psychophysiologic insomnia, patients worry about being unable to sleep and focus on their insomnia, which causes frustration and further inability to fall asleep. In delayed sleep phase syndrome, the major sleep episode is delayed in relation to the desired clock time, resulting in symptoms of sleep-onset insomnia and difficulty awakening. In advanced sleep phase syndrome, the major sleep episode is advanced in relation to the desired clock time, resulting in excessive evening sleepiness and early sleep onset, as well as awakening earlier than desired. This condition is more common in the elderly.

American Academy of Sleep Medicine. International Classification of Sleep Disorders, 2nd ed. Diagnostic and Coding Manual, Westchester, IL: American Academy of Sleep Medicine; 2005.

Bradley WG, Daroff RB, Fenichel GM, et al. Neurology in Clinical Practice, 5th ed. Philadelphia, PA: Elsevier; 2008.

98. d

Low hypocretin levels have been associated with narcolepsy, but have not been implicated in the pathophysiology of restless legs syndrome (RLS).

RLS is associated with low ferritin levels and iron deficiency, and it is thought that these may be implicated in the pathophysiology of the disease, and therefore ferritin levels should be tested in patients with this disorder. In general, iron supplementation should be initiated for ferritin levels less than 50 ng/mL. Dopaminergic pathways have been also implicated in the pathophysiology, and dopamine agonists are part of the treatment options, including pramipexole and ropinirole.

Many other conditions can also be associated with RLS, including folate deficiency, chronic renal failure, neuropathies, myelopathies, multiple sclerosis, diabetes mellitus, amyloidosis, cancer, peripheral vascular disease, rheumatoid arthritis, hypothyroidism, and certain drugs.

American Academy of Sleep Medicine. International Classification of Sleep Disorders, 2nd ed. Diagnostic and Coding Manual, Westchester, IL: American Academy of Sleep Medicine; 2005.

Bradley WG, Daroff RB, Fenichel GM, et al. Neurology in Clinical Practice, 5th ed. Philadelphia, PA: Elsevier; 2008.



If you find an error or have any questions, please email us at admin@doctorlib.org. Thank you!